ASSET A QUESTION A DAY-CLASS VIII

04/03/24

Subject: ENGLISH
Choose the option that completes the given sentence meaningfully.

The world would be a happier place ____________ people were not so selfish.
A
if only
B
provided that
C
supposing
D
although

03/03/24

Subject: ENGLISH
Choose the CORRECT option that MATCHES the sentence given below.

He was known as a great playwright and as a great artist.
A
He was not known only as a great playwright and as a great artist.
B
He was not known as a great playwright but only as a great artist.
C
He was known only as a great playwright and not as a great artist.
D
He was not only known as a great playwright but also as a great artist.

29/02/24

Subject: MATHS
Using which set of sticks can a triangle be formed?

A
B
C
D

28/02/24

Subject: SCIENCE
Nirav's science teacher wants to explain to the class how seasons occur and specifically why it is colder in winter. Which of these may be a part of the demonstration she uses?
A
Heat some water, allow the evaporated steam to come in contact with a glass containing ice cubes.
B
Take a football, a tennis ball and a table tennis ball and arrange them along an ellipse on a flat surface.
C
In a darkened room, shine a torch on a cardboard sheet, once perpendicularly and once at an angle.
D
Take the students to the playground during noon, and then to the computer room when the AC is on.c

Correct Answer: C

27/02/24

Subject: MATHS
Which of these is likely to be 5.5 METRES tall?
A
a tiger
B
a man
C
a gorilla
D
a giraffe

Correct Answer: D

26/02/24

Subject: ENGLISH
Choose the option which means the same as the given statement.

As you sow, so shall you reap.
A
Good actions will bring rewards; bad ones, punishment.
B
We must be thankful for all that we get in life.
C
Seeds should be planted at the right time.
D
We should take advantage of every opportunity we get.

Correct Answer: A

22/02/24

Subject: MATHS
Bipasha can eat 24 rasgullas in 12 minutes. Her friend Sujata takes DOUBLE the time to eat the same number.

One day they buy 24 rasgullas and start eating them. If they eat at their normal speeds till the rasgullas are over, how many rasgullas would Sujata have eaten?
A
6
B
8
C
12
D
16

21/02/24

Subject: SCIENCE
The cross section of a flower is shown below. Which letter correctly represents the part of the flower where the pollen grains are transferred?

A
A
B
B
C
C
D
D

Correct Answer: A

20/02/24

Subject: MATHS
By what number would you multiply 6.807 x 106 to get 6.807 x 1010?
A
4
B
1000
C
4000
D
10000

Correct Answer: D

19/02/24

Subject: ENGLISH
Which word in the list given below will appear second if arranged alphabetically?


A
exert
B
excess
C
exhaust
D
excite

Correct Answer: D

18/02/24

Subject: MATHS
Choose the CORRECT option that MATCHES the sentence given below.

He was known as a great playwright and as a great artist.
A
He was not known only as a great playwright and as a great artist.
B
He was not known as a great playwright but only as a great artist.
C
He was known only as a great playwright and not as a great artist.
D
He was not only known as a great playwright but also as a great artist.

17/02/24

Subject: SOCIAL STUDIES
Which of the following activities did the Brahmo Samaj focus on to bring about significant societal change in 19th century India?
A
Education of Brahmins
B
Empowerment of women
C
Revival of Vedic rites
D
Encouragement of land reforms

15/02/24

Subject: MATHS
Shivani pays Rs. 450 for a ready-made salwar kurta set she buys from Grandways during a sale (see the notice). How much money did she save because of the sale?


A
Rs. 55
B
Rs. 50
C
Rs. 49.50
D
Rs. 40.50

14/02/24

Subject: SCIENCE
Which of the following would be a chemical change?
A
A metal on heating to a high temperature turns into liquid form.
B
A metal gets a greenish coating on exposure to air.
C
A metal is drawn into wires.
D
A metal is beaten into thin sheets.

13/02/24

Subject: MATHS
(x - 2)(x - 2) + 4x is equal to
A
x2 - 4 + 4x
B
6x + 4
C
x2 + 4 + 4x
D
A metal is beaten into thin sheets.

07/02/24

Subject: SCIENCE
The United Nations Committee for the Transport of Dangerous Goods has specified symbols to be used while transporting dangerous goods.



Which of these substances would carry this sign while being transported?
A
Uranium
B
Petrol
C
LPG
D
Coal

06/02/24

Subject: MATHS
A part of quadrilateral EFGH is covered with an object X.



Which of the following is DEFINITELY true about quadrilateral EFGH?
A
It must be a parallelogram.
B
It must be a rectangle.
C
It must be a square.
D
None of the above is DEFINITELY true.

Correct Answer: D

05/02/24

Subject: ENGLISH
Which of the following is the meaning of the given expression?

Hold your horses.
A
Don't let the horses run away.
B
Be patient, slow down.
C
Do not shout at the horses.
D
Tie the horses securely to the cart.

04/02/24

Subject: ENGLISH
Choose the CORRECT option that MATCHES the sentence given below.

He was known as a great playwright and as a great artist.
A
He was not known only as a great playwright and as a great artist.
B
He was not known as a great playwright but only as a great artist.
C
He was known only as a great playwright and not as a great artist.
D
He was not only known as a great playwright but also as a great artist.

01/02/24

Subject: MATHS
Which of the following can NOT be used to express the capacity of a container?
A
dm3
B
litre
C
cm2
D
ml

31/01/24

Subject: SCIENCE
Any three colours of light, which produce white light when combined with the correct intensity, are called PRIMARY COLOURS of light. (Red, Green and Blue)

Colours produced by the addition of equal intensities of two primary colours of light are called SECONDARY COLOURS of light. (Yellow, Cyan and Magenta)

Any two colours of light, which produce white when combined, are said to be COMPLEMENTARY COLOURS of each other.

Study the figure given here and answer the following question.



Which of the following colours is a complementary colour of 'BLUE'?
A
Cyan
B
Magenta
C
Red
D
Yellow

30/01/24

Subject: MATHS


The average height of Radhika, Gopu, Ashish and Komal shown above, is 118 cm. What is the SUM of their heights?
A
156 cm
B
472 cm
C
590 cm
D
We can't say from the given information.

29/01/24

Subject: ENGLISH
Identify the sentence which has a word that rhymes with 'reel'.
A
The child ran down the stairs with her hand on the rail.
B
The fleet of ships looked lovely in the pale evening light.
C
A good doctor tries his best to heal even his sickest patient.
D
My friend looked up worriedly as she heard the door bell.

Correct Answer: C

27/01/24

Subject: SOCIAL STUDIES
Which of the following forest types are likely to be found in the Saurashtra region of Gujarat?
A
Alpine forest
B
Monsoon rain forest
C
Dry deciduous forest
D
Mangrove forest

Correct Answer: C

26/01/24

Subject: SCIENCE
For an average sized door, which of the following is closest to 1 metre?
A
Thickness
B
Width
C
Height
D
Surface area

Correct Answer: B

25/01/24

Subject: MATHS
Ananth ran a 4 km stretch at an average speed of 10 km an hour. If he started his run at 5:00 a.m., at what time did he finish?
A
5:24 a.m.
B
5:40 a.m.
C
7:25 a.m.
D
7:30 a.m.

Correct Answer: A

23/01/24

Subject: MATHS
(xy)2 × (xy)M2 is equal to
A
2xy2
B
x2y2
C
x2y4
D
x4y4

22/01/24

Subject: ENGLISH
Which of these words would appear last if arranged alphabetically?



A
please
B
plinth
C
plump
D
plastic

Correct Answer: C

20/01/24

Subject: SOCIAL STUDIES
The picture shown below represents a disaster management initiative. Which of the following is the disaster depicted in the picture?



A
Earthquake
B
Flood
C
Fire
D
Cyclone

Correct Answer: D

19/01/24

Subject: SCIENCE
There are 8 balls M, N, O, P, Q, R, S and T. 7 of them are identical, the 8th is either heavier or lighter.Only an accurate beam balance with 2 pans is available.
The result of 3 weighings is as shown:



Which is the odd ball, and is it heavier or lighter?
A
Q, heavier
B
M, lighter
C
O, lighter
D
It is not possible to tell for sure

Correct Answer: C

18/01/24

Subject: MATHS
The height of triangle PQR is double the height of triangle ABC, while the base of triangle ABC is double the base of triangle PQR.

The ratio of the areas of triangles ABC and PQR will be
A
1:1
B
1:2
C
2:1
D
2:3

17/01/24

Subject: SCIENCE
When a person's blood pressure is measured, two readings are obtained - the Systolic and the Diastolic. These two numbers are used to determine if the person has high blood pressure (hypertension).


The blood pressure readings of 4 patients is given below. Which of them is suffering from Stage 1 Hypertension?
A
A
B
B
C
C
D
D

Correct Answer: C

16/01/24

Subject: MATHS
What is 7799 ÷ 19?
A
Quotient = 41, Remainder = 0
B
Quotient = 41, Remainder = 9
C
Quotient = 41.4, Remainder = 14
D
Quotient = 410, Remainder = 9

Correct Answer: D

15/01/24

Subject: ENGLISH
The following headline appeared in a recent edition of The Times of India:



Which is the correct expansion of the given headline?
A
Saurav Ganguly has retained as the Indian skipper.
B
Saurav Ganguly was been retained as the Indian skipper.
C
Saurav Ganguly has been retained as the Indian skipper.
D
Saurav Ganguly will be the new Indian skipper.

Correct Answer: C

13/01/24

Subject: SOCIAL STUDIES
Which of the following cannot be answered from the given map?
A
Which 5 star hotel is closest to the Hussain Sagar Lake?
B
How far is the airport from the Taj Residency Hotel?
C
In which direction from the Golkonda Fort are the Qutb Shahi tombs?
D
Which tank is close to the Nehru Zoological park?

12/01/24

Subject: SCIENCE
The chart below shows the rate of heartbeat of different animals. Their weights are also given.


From the chart, which of these appears to be the relationship between the weight of an animal and its average heart rate?
A
The average heart rate of the heavier animals is more.
B
The average heart rate of the heavier animals is less.
C
The average heart rate of the light and heavy animals is less than that of the other animals.
D
There is no relationship between the weight of an animal and its average heart rate.

Correct Answer: B

11/01/24

Subject: MATHS
Nandini makes 'halwa' one evening and divides it into four equal portions for her family of four. However, just as they are about to eat it, an unexpected guest arrives and Nandini has to now re-divide the halwa into five equal portions. By what percentage has each family member's share reduced due to this?
A
5%
B
10%
C
20%
D
25%

Correct Answer: C

10/01/24

Subject: SCIENCE
The figure shows a boy throwing a javelin. For a given javelin mass, the distance the javelin travels depends upon the force exerted by the boy's arm and the time for which it is exerted.
The force exerted multiplied by the time for which it is exerted is called the impulse.

Impulse = Force x time



A
kg
B
kg.m.s2
C
kg.m/s
D
Kg.m/ s2

Correct Answer: C

09/01/24

Subject: MATHS
How many pieces exactly like this one are needed to form a complete circle (circular region)?


A
6
B
8
C
10
D
12

Correct Answer: D

08/01/24

Subject: ENGLISH
Mr. Kolhe loves to travel. He likes to plan his travel according to the alphabetical order of the places.



Which place will he go to first? Choose the correct answer from the list given in the box above.
A
Bangalore
B
Bangripost
C
Bangkok
D
Bangladesh

Correct Answer: A

06/01/24

Subject: SOCIAL STUDIES
The cartoon shown here is related to a natural disaster. Which one?



A
Tsunami
B
Cyclone
C
Flood
D
Cloud burst

05/01/24

Subject: SCIENCE
The 'Flame Test' is used in the identification of certain metals. It is based on the observation that light emitted by any element gives a unique spectrum when passed through a spectroscope. When a salt of the metal is introduced into a Bunsen burner flame, the metallic ion produces characteristic colour in the flame. Some metals and the colours they produce are:



White light can be considered the sum of RED, BLUE and GREEN coloured light. In other words, if all RED was absorbed from WHITE light, it would appear to be CYAN. That is:

CYAN = GREEN + BLUE
MAGENTA = RED + BLUE
YELLOW = RED + GREEN

The colour of the flame produced by sodium ion masks many other coloured flames. What would be the best filter to use in order to block the colour of sodium flame?
A
Red
B
Yellow
C
Blue
D
Plane glass

04/01/24

Subject: MATHS
What number should come in the box to make the following statement true?
34 + 32 = 9 × ___
A
4
B
7
C
10
D
81

01/01/24

Subject: ENGLISH
Choose the suffix that could be added to the base word 'fresh'.
A
-ing
B
-ed
C
-ment
D
-ness

30/12/24

Subject: SOCIAL STUDIES
Debasish wants to apply for a loan to UTI bank for house construction. He is asked by the bank authority to submit a document showing PROOF OF IDENTITY. Which of these documents can Debasish produce for this?
A
latest electricity bill
B
latest salary slip
C
his passport
D
the UTI bank cheque book

29/12/24

Subject: SCIENCE
Which of the following joints would allow rotational motion about MORE THAN one axis?
A
B
C
D
All of these

28/12/24

Subject: MATHS
The integer that is 25 more than (-435) is
A
-450
B
-460
C
-410
D
460

27/12/24

Subject: SCIENCE
A hydro carbon C4H10 has a boiling point of 00 C. Assuming it is stable at 25oC, what would be its state at that temperature(all other conditions like pressure remaining the same)?
A
Solid
B
Liquid
C
Gas
D
(Cannot predict without knowing the melting point)

26/12/24

Subject: MATHS
Gina has a brief case which she can lock using a 3-digit code using any of the digits 1, 2, 3 and 4.



How many different 3-digit codes can she form using these four digits?
A
3 × 4
B
4 × 4 × 4
C
3 × 3 × 3 × 3
D
3 + 3 + 3

25/12/24

Subject: ENGLISH
Which word best completes the sentence?

The edges of the fresh sheets of paper were as sharp as________.
A
razors
B
rulers
C
the mind
D
wall paper

23/12/24

Subject: SOCIAL STUDIES
The map shown below is a route starting from Chennai to attract foreign tourists to visit ______



If they had to choose the brand with the longest expected life based on the above experiments, they should choose _________.
A
Brand P
B
Brand Q
C
Brand R
D
Brand T

22/12/24

Subject: SCIENCE
Five friends, Lata, Mala, Shaila, Rohit and Ali each tried out two of the five different brands of batteries to check which brand lasted the longest. The data collected is shown below.



If they had to choose the brand with the longest expected life based on the above experiments, they should choose _________.
A
Brand P
B
Brand Q
C
Brand R
D
Brand T

21/12/23

Subject: MATHS
In a circular cricket stadium, the seats are arranged around the main playground, in concentric circles. The innermost circle (nearest to the playground) has 100 seats and the outermost circle has 200 seats, with the number of seats increasing uniformly per circular row. If there are 26 circular rows of seats in all, how many seats are there in the 20th row?
A
150
B
172
C
176
D
180

20/12/23

Subject: SCIENCE
A patient with cancer of the food pipe, cannot eat and swallow. So to feed him, a tube is inserted directly into his stomach. Properly mashed food is now given to the patient directly through the pipe.
Now that the food is going directly to the stomach instead of through the food pipe, which of these is likely to happen?
A
water will not be absorbed
B
the kidney will stop working
C
carbohydrates will not be digested
D
(no major problem will arise)

19/12/23

Subject: MATHS
In a magic square, the sum of every row, column and diagonal is the same. Shown below is a 3 x 3 magic square:



What is the value of 10x?
A
60
B
40
C
30
D
10

18/12/23

Subject: ENGLISH
Choose the best words to complete the sentence below.

She told her friend that if he ________ his promise, she ________ speak to him again.
A
break - would never
B
break - will never
C
broke - will never
D
broke - would never

16/12/23

Subject: SOCIAL STUDIES
The illustration shown below is based on an artists's perception of a recent rail accident in Bihar. Observe the illustration and read the text carefully. According to this, who was primarily responsible for the accident?



A
the engineers of the British East India Company
B
the engineers of modern Indian Railways
C
the construction workers
D
the train driver

15/12/23

Subject: SCIENCE
Raju has two batteries. One is a 9V battery and the one is a regular 1.5V pencil cell. Each battery has a + and - noted on it as shown.
How can Raju connect the two batteries together in a simple circuit?

A
B
C
D

14/12/23

Subject: MATHS
Cube X shown below has a different number (from 1 to 6)on each face.



Which of the views shown below can NOT be of cube X?

A
View 1
B
View 2
C
View 3
D
All the views are of cube X

13/12/23

Subject: SCIENCE
A fine copper gauze is taken and placed about half a centimeter above a Bunsen Burner. When the gas is switched on and lighted above the gauze, the flame burns only above the copper gauze as shown in the figure. This demonstrates that:



A
Copper is a very good conductor of heat.
B
Copper is a very poor conductor of heat.
C
The wire gauze blocks the flow of oxygen.
D
The wire gauze blocks the flow of the gas.

12/12/23

Subject: MATHS
An equilateral triangle TSR is to be drawn on base SR of square PQRS.

Where will point T lie?
A
inside square PQRS
B
on the midpoint of side PQ
C
exactly on point P
D
outside the square, over the top of PQ

05/12/23

Subject: MATHS
Qadir wanted to form a triangle with exactly 12 identical matchsticks, without bending or breaking any. He makes one side of the triangle with 6 matches.


How many matchsticks should he use for the other two sides (respectively)?
A
3 matchsticks, 3 matchsticks
B
4 matchsticks, 2 matchsticks
C
5 matchsticks, 1 matchstick
D
He will not be able to make a triangle.

04/12/23

Subject: ENGLISH
Which word best completes the sentence?

The cut needed _________.
A
bandageing
B
bandaging
C
bandeging
D
bandajing

02/12/23

Subject: SOCIAL STUDIES
The global presence of Tata Steel of India is depicted in the map given below. Which of the following is TRUE about Tata Steel with relation to the map?



A
Tata Steel has established steel plants all across Asia and Africa.
B
Tata Steel recently acquired a steel plant in America.
C
Tata Steel has customers in Asia, Africa, Europe and North America.
D
Bangladesh is the main centre of raw material extraction for Tata Steel.

01/12/23

Subject: SCIENCE
A hydrometer is an instrument used to measure the density of liquids. Its markings are not linear. A hydrometer, whose markings have got erased, is placed in three vessels containing petrol (density 750 g/cc), water (density 1000 g/cc) and glycerine (density 1250 g/cc).



Which of these could be the correct markings on the hydrometer?
A
B
C
D

30/11/23

Subject: MATHS
A bakery makes bread loaves in two sizes as shown below.



The smaller loaf is priced at Rs.10 and the larger at Rs. 20. Which loaf is better (more economical) to buy?
A
the smaller one
B
the larger one
C
both are equally economical
D
depends on the number of slices they are cut into

28/11/23

Subject: MATHS
The jug below has to be filled with a liquid, using the beaker shown.



If the beaker is to be filled with exactly as much liquid as is required to fill the jug upto the 2 litre mark, what part of it should be filled?
A
1⁄4
B
1⁄2
C
2⁄3
D
3⁄4

27/11/23

Subject: ENGLISH
Choose the phrase that could replace the underlined words in the sentence.

Beena jumped at the chance when she was offered a ride in the fast car.
A
was frightened
B
fell down
C
was quick to say 'yes'
D
said 'no'

25/11/23

Subject: SOCIAL STUDIES
One of India's evolving neo-classical dance forms is Satriya, which has its origin in the island of Majuli in the middle of Brahamaputra River in Assam. The name Satriya is derived from Satra, a monastic institution of the Vaishnava sect.

It is likely that the Satriya dance is based on events in the life of:

A
Lord Buddha
B
An Ahom King
C
Lord Krishna
D
Lord Shiva

24/11/23

Subject: SCIENCE
Crude oil is usually not used as a fuel. Which among these is NOT a reason for this?
A
Crude oil is a resource from which many other fuels can be extracted.
B
Naphtha, got from crude oil, is a raw material for many important chemicals.
C
Crude oil is a non renewable resource and thus has to be conserved.
D
Crude oil does not burn completely and produces a very smoky flame.

23/11/23

Subject: MATHS
John bought a motorcycle that advertises its mileage as being 90 km/litre, but the actual mileage is 20% less than that. If John fills 2 litres of petrol in his motorcycle, how many kilometres can he travel before the petrol gets over?
A
90 km
B
144 km
C
80 km
D
216 km

22/11/23

Subject: SCIENCE
Normal rain water has a pH of 5.6. Fishes and other aquatic animals die when the water becomes more acidic. Which of these gases is likely to make rain water more acidic?

" style="cursor: pointer; outline: 0px; max-width: 100%;" tabindex="0" />
A
Ozone
B
Sulphur dioxide
C
Nitrogen
D
Hydrogen peroxide

21/11/23

Subject: MATHS
3 unit cubes that were joined as shown in Figure 1 were taken apart and then joined again as shown in Figure 2.



Which of the following changed due to the reshaping of the solid?
A
the volume
B
the total surface area
C
both volume and total surface area
D
neither volume nor total surface area

19/11/23

Subject: SOCIAL STUDIES
One of the most severe famines of the 20th century was the Bengal famine of 1943 in which over five million people died. Prof. Amartya Sen, who was awarded a Nobel Prize for his studies of the Bengal famine, found that the pressure of war and the inability of the British to provide enough relief were the two main causes of the famine.

Which waris being referred to?

A
Sepoy Mutiny
B
World War I
C
World War II
D
Bengal Partition

17/11/23

Subject: SCIENCE
The diagram shows the mass and dimensions of three cuboids. Which one of the following statements is TRUE?

A
The blocks all have the same density
B
Blocks 2 and 3 are twice as dense as Block 1.
C
Block 1 is one-third as dense as Block 3
D
All three blocks have different densities

Correct Answer: C

15/11/23

Subject: SCIENCE
In the figure given, compared to wave P, wave Q would be _________

A
louder
B
softer (less sound)
C
of higher pitch
D
of lower pitch

14/11/23

Subject: MATHS
If the area of the figure drawn on the square grid below is 36 cm2, what will be the value of 'x'?


A
4.5 cm
B
3.6 cm
C
3 cm
D
2 cm

Correct Answer: D

13/11/23

Subject: EGLISH
Select the option which best completes the sentence below.

The head of the school is usually known as the ________.
A
Principle
B
Principal
C
Princepal
D
Prinsipal

Correct Answer: B

11/11/23

Subject: SOCIAL STUDIES
Uttarakhand is a mountainous, land-locked state in North India. It has few mineral deposits. Yet in recent years, it has been able to attract massive investments in IT, pharmaceuticals, manufacturing, etc. This is probably because of
A
the conducive climatic conditions of the state.
B
the lure of tourist and religious places.
C
the increased tax benefits provided by the government.
D
the presence of a coalition government in the state.

Correct Answer: C

10/11/23

Subject: SCIENCE
BIOLOGICAL CONTROL is term used to describe the use of living organisms, such as predators, parasitoids, and pathogens, to control pest insects, weeds or diseases.

An example of biological control is using ______________________.
A
a cat to kill mice at home
B
pesticides to kill insects
C
medicine to poison food of the pest
D
scarecrows to scare away birds

Correct Answer: A

09/11/23

Subject: MATHS
How many pairs of parallel lines are there in the figure below?


A
one
B
two
C
three
D
six

Correct Answer: B

08/11/23

Subject: SCIENCE
If you took all of the atoms out of a chair, what would be left?
A
The chair would still be there, but it would weigh less.
B
The chair would be exactly the same as before.
C
There would be nothing left of the chair.
D
A pool of liquid would be left in place of the chair.

Correct Answer: C

07/11/23

Subject: MATHS
A 'micrometre' ( µm) is a unit for measuring very small lengths, where
1 µm = 1 × 10 − 3 mm.


What is the diameter(in mm) of the cell shown above?
A
0.00078
B
0.0078
C
0.26
D
7800

Correct Answer: B

06/11/23

Subject: ENGLISH
Which words will best complete the following sentence?

The world would be a happier place ____________ people were not so selfish.
A
if only
B
provided that
C
supposing
D
although

Correct Answer: A

04/11/23

Subject: SOCIAL STUDIES
The cartoon shown below is related to an encroachment free drive by the state government of Delhi. What could be the possible reason behind this drive being carried out?

A
to rehablitate people affected by cold wave
B
for widening a public road
C
to remove the people living below poverty line
D
to control terrorist activity

Correct Answer: B

03/11/23

Subject: SCIENCE
Four students, P, Q, R and S are standing in front of a mirror facing it. Their positions, as seen from the top, are shown below. Which of these students can see Q as well as Q's reflection in the mirror?



A
only P
B
only R and S
C
only R
D
only S

Correct Answer: B

02/11/23

Subject: MATHS
A square wire frame is cut open and then bent to form a regular hexagon. Each side of the square as well as of the hexagon formed is a whole number (when measured in cm). Which of these could be the perimeter of the hexagon?
A
26 cm
B
45 cm
C
66 cm
D
96 cm

Correct Answer: D

01/11/23

Subject: SCIENCE
Which of these best explains why a chullah fire (or a camp fire) burns brighter when it is fanned?



A
the air tries to put out the fire, so the fire burns brighter
B
the amount of fuel available to the fire increases
C
the amount of oxygen available to burn increases
D
it removes the burnt ashes allowing the wood to burn

Correct Answer: C

31/10/23

Subject: MATHS
Five numbers, when arranged in ascending order, form a series in which the difference between each number and the number next to it is 7. If 8 and 36 are two of those numbers, which one of the following could be one of the other three numbers?
A
14
B
27
C
29
D
35

Correct Answer: C

30/10/23

Subject: ENGLISH
Which expression can best fill the blank?

This fight has been going on for years. Why don't you two ______________ and be friends?
A
chip in
B
do the honours
C
call it quits
D
resign

Correct Answer: C

28/10/23

Subject: SOCIAL STUDIES
Kutch in Gujarat has traditionally had a pastoral economy. Which of these geographical features of Kutch has contributed to such an economic tradition?
A
its Banni grassland
B
its long coastline
C
its bountiful lakes
D
its low rocky hills

Correct Answer: A

27/10/23

Subject: SCIENCE


Three identical balls are taken at three different places P, Q and R on the earth as shown in the diagram. When the balls are dropped, gravity makes them fall.
Which of the following diagrams best shows the directions in which the balls will fall at P, Q and R?
A
B
C
D

Correct Answer: C

26/10/23

Subject: MATHS
According to the Flag Code of India, the length to width ratio of our National Flag should be 3 : 2. The flag should be made up of three rectangular sub-panels of equal width, with the top panel saffron, bottom panel green and the middle panel white, having the Ashok Chakra at its centre.



What is the length to the width ratio of each sub-panel in the flag?
A
9 : 2
B
9 : 4
C
3 : 2
D
3 : 6

Correct Answer: A

25/10/23

Subject: SCIENCE
Study the simple circuit shown here. A compass is placed near the circuit as shown. Which of these statements is TRUE?



A
If the battery terminals are connected the other way, the compass needle will deflect in the other direction.
B
The bulb will light up only when the battery terminals are connected in one particular way, not the other.
C
There can be situations when the compass needle may deflect, but the bulb does not light up or vice versa.
D
The extent of deflection of the compass needle will change if it moved along the wire from battery to bulb

Correct Answer: A

24/10/23

Subject: MATHS
Look at the hands of the clock shown here:



How much time passes before the MINUTE hand of this clock reaches the position where the hour hand is at present?
A
45 minutes
B
45 minutes 15 seconds
C
30 minutes
D
30 minutes 75 seconds

Correct Answer: D

23/10/23

Subject: ENGLISH
Select the option which best completes the sentence below.

A person or animal in a story is ___________.
A
a theme
B
a character
C
a conclusion
D
a chapter

Correct Answer: B

20/10/23

Subject: SCIENCE
Corn is used as food and also as fodder for animals. In recent years, however, corn bushels are being used to produce Ethanol, an important fuel. Which of these correctly compares the use of corn as FUEL and FOOD?


A
B
C
D

19/10/23

Subject: MATHS
The length of the worm shown next to the ruler below is about


A
7.5 cm
B
10 cm
C
15 cm
D
22.5 cm

Correct Answer: B

18/10/23

Subject: SCIENCE
Shakuntala takes some amount of water in a measuring cylinder and reads the value keeping her eye at the level of water. Then she adds a little more water. For measuring the new level of water, she should


A
keep her eye at the same level
B
raise the level of her eye
C
lower the level of her eye
D
(Re-read the first level correctly)

Correct Answer: B

17/10/23

Subject: MATHS
Two numbers whose sum is 0 are called 'opposite' numbers.
According to this, what number would be the 'opposite' of − 2x
A
2x
B
-x⁄2
C
x⁄2
D
2 + x

Correct Answer: A

16/10/23

Subject: ENGLISH
What do you say in a shop if you only want to look around but not buy anything?
A
I'm just browsing.
B
I'm just shoplifting.
C
I'm just viewing things.
D
I'm just waiting.

Correct Answer: A

15/10/23

Subject: SOCIAL STUDIES
Devpal wants to open an account in a nationalised bank, and get the highest possible return on his money. For this, he would have to put his money into
A
a savings account
B
a current account
C
a fixed deposit
D
a loan account

Correct Answer: C

13/10/23

Subject: SCIENCE
The energy transformations shown in the figure are seen in which of these?


A
kerosene in a lamp
B
battery in a torch
C
coal in a fire place
D
petrol in a car

Correct Answer: D

12/10/23

Subject: MATHS
Which of these could have a value of 120%?
A
the increase in the number of students in a school during a year.
B
the marks scored by a student in a test of 500 marks.
C
the discount on a music system priced at Rs. 18000.
D
the decrease in the number of polio cases in a particular district.

Correct Answer : A

11/10/23

Subject: SCIENCE
The Wildlife Institute of India survey reveals that 70-80% of individual freshwater marshes and lakes in the Gangetic plains have been lost in the last five decades. At present only 50% of India's wetlands remain.
Which of the following is a NOT a cause for the disappearance of wetlands?
A
Increasing urbanization and industrialisation
B
Increased tree-cutting and deforestation
C
Increase in migratory bird population
D
Recovery of marsh land for agriculture

Correct Answer: C

10/10/23

Subject: MATHS
Numbers that can be written in the form m⁄n where m and n are integers and n is not equal to 0 are called rational numbers.

Which of the numbers in the list below are rational numbers? -12⁄5, 12.5, 12
A
12 and -12⁄5
B
Only -12⁄5
C
Only 12
D
Only 12

Correct Answer: D

09/10/23

Subject: ENGLISH
Select the option which best completes the sentence below.
You seem to be having trouble there. ______ I help you?
A
Would
B
Will
C
Shall
D
Do

Correct Answer: C

07/10/23

Subject: SOCIAL STUDIES
The poster shown here is drawn by the well-known cartoonist Mario Miranda. It is meant to

A
promote export of Goan seafood.
B
promote cottage industies in Goa.
C
promote tourism industry in Goa.
D
conserve rare wildlife in Goa.

Correct Answer:C

06/10/23

Subject: SCIENCE
What is the MAIN advantage of having two ears rather than only one to hear with?
A
What is the MAIN advantage of having two ears rather than only one to hear with?
B
The second ear is a backup in case the first ear fails
C
The direction of sounds can be judged with two ears.
D
The symmetry of the human face is maintained.

Correct Answer: C

05/10/23

Subject: MATHS
The table below shows the minimum temperatures (in oC ) on different days of a week in Leh.



On which day of the week was the lowest temperature recorded in Leh?
A
Dec 18
B
Dec 19
C
Dec 21
D
Dec 22

Correct Answer: D

04/10/23

Subject: SCIENCE
We know that all matter consists of molecules. When a gas is compressed, what happens to its molecules?
A
Nothing happens to the molecules, their arrangement or their movement.
B
Each of the molecules gets compressed by about the same amount.
C
Nothing happens to the molecules, but the space between them reduces.
D
Nothing happens to the molecules or their arrangement, but they start moving faster.

Correct Answer: C

03/10/23

Subject: MATHS
What will be the value of 3 + 2 × (10 − 4) ÷ 2 − 1?
A
8
B
14
C
15
D
30

Correct Answer: A

02/10/23

Subject: ENGLISH
Which of these words will take the second place if they are arranged alphabetically?

A
radio
B
radish
C
radical
D
radiant

Correct Answer: C

30/09/23

Subject: SOCIAL STUDIES
The figure shown here is a mode of transport pulled by dogs and popular in a specific geographical region. Identify this region.
" style="cursor: pointer; outline: 0px; max-width: 100%;" tabindex="0" />
A
rainforests of Brazil
B
tundra region of Finland
C
Gobi desert of Mongolia
D
fertile plains of Bangladesh

Correct Answer: B

29/09/23

Subject: SCIENCE
Four test tubes are taken with water and a special chemical which can detect the presence of carbon dioxide. A water plant and / or water animal was taken in each test tube as shown.

(The chemical mentioned is not harmful to the plant or animal.)

The chemical causes the solution to turn yellow if there is a lot of carbon dioxide. Which test tube is most likely to turn yellow?
A
B
C
D

Correct Answer: B

28/09/23

Subject: MATHS
If the length of the second's hand of a watch (having a circular dial) is halved, the angle through which it moves in one second will
A
be doubled
B
become one fourth
C
be halved
D
remain unchanged

Correct Answer: D

27/09/23

Subject: SCIENCE
The arrangement shown below was used in 19th century theatres to create images of ghosts that could appear and disappear at will.



What should be placed at X for the trick to work?
A
a light source
B
a mirror
C
another actor
D
a lens

Correct Answer: A

26/09/23

Subject: MATHS
The sum of all the whole numbers from 1 to 45 is subtracted from the sum of all the whole numbers from 1 to 50 as shown:
(1 + 2 +....+ 50) - (1 + 2 +.....+ 45)
What will be the result?
A
5
B
50
C
240
D
285

Correct Answer: C

25/09/23

Subject: ENGLISH
Identify the sentence with a rhyming word for 'broad'.
A
The brood of chickens followed the mother hen everywhere.
B
The thief tied the man's hands with a nylon cord.
C
If they have no bread, let them eat cake.
D
The toad sang its merry song unaware of the coarseness of its sound.

Correct Answer: B

23/09/23

Subject: ENGLISH
Choose the CORRECT option that MATCHES the sentence given below.

He was known as a great playwright and as a great artist.
A
He was not known only as a great playwright and as a great artist.
B
He was not known as a great playwright but only as a great artist.
C
He was known only as a great playwright and not as a great artist.
D
He was not only known as a great playwright but also as a great artist.

22/09/23

Subject: SCIENCE
The cartoon here tries to portray



A
WIPRO's entry into the global market
B
diversification of WIPRO into different businesses
C
merging of smaller IT companies with WIPRO
D
WIPRO's concern for environmental protection

21/09/23

Subject: MATHS
A pizza is divided into equal slices as shown. What part of the pizza below would remain if you ate two of the slices?



A
40%
B
60%
C
66.7%
D
77.5%

20/09/23

Subject: SCIENCE
The diagram here shows a mountain of height 2000 metres. The direction of the wind and some air temperatures are indicated.



From this information, it is very likely that the region at X will be a
A
dry region
B
jungle
C
large lake
D
Plateau

Correct Answer: A

19/09/23

Subject: MATHS
Which of the following numbers is equal to 25?
A
22 × 23
B
22 + 23
C
210 − 25
D
220 ÷ 24

Correct Answer: A

18/09/23

Subject: ENGLISH
Choose the option which best completes the sentence.

The computational speed of computers ________ in the last few years.
A
have doubled
B
has doubled
C
had doubled
D
is doubling

Correct Answer : B

16/09/23

Subject: SOCIAL STUDIES
In ancient Greek society, social hierarchy played an important role in day-to-day functioning. In the absence of literary records, what evidence might archaeologists use to understand different levels of hierarchy in ancient Greek society?
A
differences in grains consumed.
B
differences in offerings found at graves.
C
differences in religious beliefs.
D
differences in human skeletons found.

Correct Answer: B

15/09/23

Subject: SCIENCE
The graph given shows the population of two species ladybirds and aphids over time. Which of these appears to be a possibility based on this graph?



A
Ladybirds probably feed on aphids
B
Aphids probably feed on ladybirds
C
Ladybirds and aphids do not feed on each other
D
Nothing can be said about them based on this

Correct Answer: A

14/09/23

Subject: MATHS
The plan of a shopping plaza is shown below. During rush hour, people have to enter the plaza through entrance E and on coming to an intersection they can move only in the directions indicated by the arrows. At each intersection, an equal number of people take each available path. If there is only one path at an intersection, everyone proceeds through that.


If 400 people enter the plaza through entrance E, how many people would come out from exit H?
A
325
B
250
C
200
D
133

Correct Answer: A

13/09/23

Subject: SCIENCE
On a hot summer day, Sarita decides to move her fridge to her small bedroom, keep its door open, and lock all the doors of the room. She is hoping that the fridge will cool the room like an AC. Will her plan work?
A
Yes, because the room is small and its doors are closed.
B
Yes, but it will take more time compared to an AC.
C
No, because the heat is being returned to the room.
D
No, because a fridge does not cool when its door is open.

Correct Answer: C

12/09/23

Subject: MATHS
Five identical rectangular pieces of cloth of different prints when arranged as shown form a square patch. If the perimeter of each rectangular piece is 48 cm, what is the perimeter of the square patch?


A
240 cm
B
120 cm
C
96 cm
D
80 cm

Correct Answer: D

11/09/23

Subject: ENGLISH
Which of these sentences has no mistake?
A
He said, ''You are correct.''
B
He said, ''you are correct''.
C
He said ''you are correct''.
D
He said, ''You, are correct.''

Correct Answer: A

10/09/23

Subject: SOCIAL STUDIES
Study this poster carefully. What is the MOST APPROPRIATE message to go into the blank space on the bottom right part of the poster?

A
It is exactly one-fourth of 90o, the total number of degrees of latitude.
B
The earth's axis of rotation is tilted by that much - 23.5o
C
The limit has been agreed to by all the nations of the world.
D
There is no reason, and the limit has been fixed arbitrarily.

08/09/23

Subject: SCIENCE
The Tropic of Cancer at 23.5oN latitude marks the northern limit of the places on earth where the Sun is ever exactly overhead. Why is this limit at 23.5o and not some other latitude?
A
It is exactly one-fourth of 90o, the total number of degrees of latitude.
B
The earth's axis of rotation is tilted by that much - 23.5o
C
The limit has been agreed to by all the nations of the world.
D
There is no reason, and the limit has been fixed arbitrarily.

Correct Answer: B

07/09/23

Subject: MATHS
A corner of a square paper is folded along the dotted line as shown below.



After folding, if
A
700
B
550
C
350
D
200

Correct Answer: B

06/09/23

Subject: SCIENCE
Study this circuit which uses two 2-way switches. Unlike a regular switch, a 2-way switch is connected to a circuit even when it is in the 'off' position.


The special feature of this circuit is that:
A
the bulb can be switched on only if both the switches are on.
B
the bulb can be switched on by any one of the two switches.
C
one switch can only turn the bulb on, and the other can only turn it off.
D
multiple bulbs can be controlled through multiple switches.

Correct Answer: B

05/09/23

Subject: MATHS
A geometrical game has 4 wooden rods of lengths 1, 2, 3 and 4 units respectively. How many different triangles is it possible to make from these rods using only three rods at a time?



A
1
B
2
C
3
D
4

Correct Answer: A

04/09/23

Subject: ENGLISH
Choose the option which best completes the sentence.

It is better to prepare oneself well rather than go for a job without __________.
A
preparatory
B
preparation
C
prepared
D
prepares

Correct Answer: B

03/09/23

Subject: SOCIAL STUDIES
Which of these business or financial organizations can NOT be listed on the National Stock Exchange of India?
A
Reserve Bank of India
B
Reliance Industries Limited
C
ICICI Bank Limited
D
ICICI Bank Limited

Correct Answer: A

03/09/23

Subject: SCIENCE
June 21/22 of any year is ________________
A
when day and night are of equal duration throughout the world
B
the Winter Solstice in the Southern Hemisphere.
C
the longest day in all parts of the world.
D
the shortest day in all parts of the world.

31/08/23

Subject: MATHS
The average height of 3 boys is 130 cm. If the shortest boy is 120 cm tall, how tall could the tallest boy be?
A
180 cm
B
165 cm
C
145 cm
D
130 cm

Correct Answer: C

30/08/23

Subject: SCIENCE
In a car battery, plates of lead and lead dioxide are immersed in a liquid, L. A chemical reaction occurs leading to the production of lead sulphate and water. What kind of a liquid is L likely to be?
A
An acid
B
A base
C
Salt solution
D
Distilled water

Correct Answer: A

29/08/23

Subject: MATHS
Ami is playing with small identical cubes. She joined 8 of them to form a cube as shown.


At least how many more of the small cubes will she need to join to this formation to get the next sized cube?
A
9
B
12
C
19
D
27

Correct Answer: C

28/08/23

Subject: ENGLISH
Choose the option which best completes the sentence.

Sandpaper feels ______ when touched.
A
course
B
coarse
C
cores
D
cause

Correct Answer: B

26/08/23

Subject: SOCIAL STUDIES
In the Patachitra style, human figures are portrayed with extended beak-like noses, elongated eyes, prominent chins and finely detailed clothes and jewellery.

Which of the following is also in the patachitra style?
A
B
C
D

Correct Answer: B

25/08/23

Subject: SCIENCE
The frequency range in which some common animals can hear is shown in the graphic below.



A manufacturer wants to create a pest controller that will emit loud sounds that can be heard by insects and rodents (and will hence drive them away), but will not affect humans and pets.
What frequencies of sound should the pest controller produce?
A
Below 20 hertz
B
100 hertz - 1,000 hertz
C
40,000 hertz - 100,000 hertz
D
Above 100,000 hertz

Correct Answer: C

24/08/23

Subject: MATHS
A cuboidal tank was filled with water to the level shown below.



After 50 more litres of water is poured into the tank, it was 3⁄4 full. What is the capacity of the tank?
A
120 litres
B
110 litres
C
100 litres
D
90 litres

Correct Answer: A

23/08/23

Subject: SCIENCE
A gas is taken in a vessel which is then sealed. The gas is then heated. How will the temperature, pressure and volume of the gas change?
A
Since the vessel is sealed, all of them will remain the same.
B
The temperature will increase, the pressure and volume will remain the same.
C
The temperature and pressure will increase, the volume will remain the same.
D
The temperature, pressure and volume will all increase.

Correct Answer: C

22/08/23

Subject: MATHS
There is a certain relation between the corresponding members of X and Y in the following figure.



Which of the following equations describes the relation correctly?
A
x = y + 1
B
y = 2x -1
C
x = 2y -1
D
y = 2x2-1

Correct Answer: D

21/08/23

Subject: ENGLISH
Which of these best completes the sentence below?

My friend was in search of some reasonable _____________.
A
acomodation
B
accommodation
C
accomodation
D
ackomodation

Correct Answer: B

19/08/23

Subject: SOCIAL STUDIES
The paintings shown here are drawn by the folk artists of Eastern India. These paintings are meant to generate awareness among people about a deadly disease. Which one?



A
Polio
B
Leprosy
C
AIDS
D
Diarrhea

Correct Answer: C

18/08/23

Subject: SCIENCE
When a cell was examined under a microscope, it was found that it did not have a cell wall.
Which of these is also likely to be missing from the cell?
A
Nucleus
B
Mitochondrion
C
Chloroplasts
D
Vacuole

Correct Answer: C

17/08/23

Subject: MATHS
Which of the following successive discounts on an article with a marked price of M would result in the greatest benefit to the buyer?
A
15% off on M, then 25% on the reduced amount
B
25% off on M, then 15% on the reduced amount
C
40% off on M
D
All would result in the same discount

Correct Answer: C

16/08/23

Subject: SCIENCE
Contour lines on a map connect points on the surface that have the same elevation (height) above sea level.



What can be said about the flow of the river in this map?
A
It flows in a south-east direction
B
It flows in a north-west direction
C
It flows in a south-west direction
D
We cannot be sure of the flow direction.

Correct Answer: A

15/08/23

Subject: MATHS
A solid cube of side 8 cm has a black border of uniform width 1 cm painted on each of its faces.



What part of the cube's surface is painted black?
A
7⁄8
B
7⁄16
C
9⁄16
D
15⁄64

Correct Answer: B

14/08/23

Subject: ENGLISH
Which of these best completes the sentence below?

When the car is standing still, it is _____________.
A
stationery
B
stationary
C
stetionary
D
stationnary

Correct Answer: B

12/08/23

Subject: SOCIAL STUDIES
The Chola empire in South India (around 1000 AD) was known for its system of decentralisation of power. In this system, democratic methods were used to resolve small conflicts. Which of these may have been used to resolve a dispute between two farmers?
A
in the presence of the council of ministers in the Chola royal court
B
at the house of the most wealthy and powerful person in the village
C
at the village ur or sabha, the council of villagers.
D
at the house of the village priest

Correct Answer: C

11/08/23

Subject: SCIENCE
Two balls P and Q simultaneously hit a stationary ball R. The direction in which P was moving when it hit R, and the final direction in which R moves are shown.


Which of these could be the direction in which Q was moving when it hit R?
A
B
C
D

Correct Answer: D

10/08/23

Subject: MATHS
Vaishali saves Rs. 35 every week to buy a game costing Rs. 510.

Which expression below shows the amount left for her to save after n weeks?
A
(510 - 35)n
B
35n - 510
C
510 - 35n
D
(510 + 35)n

Correct Answer: C

09/08/23

Subject: SCIENCE
Hydrated copper sulphate is a blue crystalline solid. When heated, it changes to anhydrous copper sulphate, a white powder. Which of these occurs in that process?
A
Loss of water
B
Absorption of water
C
Loss of oxygen
D
Oxidation

Correct Answer: A

08/08/23

Subject: MATHS
Two identical triangular pieces of paper are pasted one on top of the other such that they overlap in a square of side 4 cm as shown below.


The resulting shape covers an area of
A
136 cm²
B
120 cm²
C
104 cm²
D
88 cm²

Correct Answer: C

07/08/23

Subject: ENGLISH
Choose the answer which means the same as the given statement.

All that glitters is not gold.
A
Everything that shines looks as good as gold.
B
Something that looks very good may not be so great.
C
There are more ways to the wood than one.
D
All's well that ends well.

Correct Answer: B

05/08/23

Subject: SCIENCE
British colonial records, oral histories and some material culture of the Apatani tribe suggest that they came to their present settlements in Arunachal Pradesh after years of migration.
What could be the main reason for this?



A
Setting up of large scale industries displaced them from time to time.
B
The search for new forest areas forced them to migrate from time to time.
C
Prolonged war with neighbouring China caused a series of migrations.
D
Fear of wildlife caused migration into valleys from forests.

Correct Answer: B

04/08/23

Subject: SCIENCE
Arrange these steps of the Scientific Method in the correct sequence:



A
2 -- 4 -- 3 -- 5 -- 1
B
3-- 4 -- 2-- 5 -- 1
C
4-- 3-- 1-- 2 -- 5
D
There isn't 1 correct sequence - many are possible

Correct Answer: B

03/08/23

Subject: MATHS
A tailor, when asked to stitch dresses for a school's annual programme, says he would stitch 25 dresses a day and finish the work in 20 days. If the school requests him to finish and deliver the dresses 5 days earlier, at least how many extra dresses would he need to stitch every day?
A
13
B
9
C
8
D
5

Correct Answer: B

31/07/23

Subject: ENGLISH
Choose the CORRECT option to complete the given sentence.
By pulling the wool over his mum's eyes, Shane was __________
A
covering her face.
B
stealing things from her
C
hiding the truth from her.
D
speaking non-stop.

29/07/23

Subject: SOCIAL STUDIES
Arrange the places: Cochin, Pune, Delhi and Nainital according to their heights above the sea level - from the lowest to the highest.
A
Cochin – Pune – Delhi – Nainital
B
Pune – Nainital – Cochin – Delhi
C
Cochin – Delhi – Pune – Nainital
D
Delhi – Cochin – Pune – Nainital

28/07/23

Subject: SCIENCE
Sheela was asked to write about an animal by her teacher. Here is what she wrote.

"On a rainy day I saw a number of crawling worm-like creatures on my lawn. They were brown. I could not see any legs. Their bodies were divided into rings. I could not even make out the head from the tail."

Identify the animals.
A
B
C
D

27/07/23

Subject: SCIENCE
Speech is much better developed in humans than in other animals. This is MAINLY due to which organ being better developed in humans?
A
Mouth
B
Tongue
C
Heart
D
Brain

26/07/23

Subject: MATHS
Which of the following is equal to 132?
A
137- 135
B
136/134
C
122 + 12
D
133/135

24/07/23

Subject: ENGLISH
Choose the option with the CORRECT spelling to complete the given sentence.

A treeless grassy plain is called a __________.
A
preirie
B
priarie
C
prairi
D
prairie

23/07/23

Subject: SOCIAL STUDIES
In recent years, there have been conflicts between various Indian states about owning or using a common natural resource. Which one?
A
river water
B
sea water
C
minerals
D
forest produces

21/07/23

Subject: SCIENCE
When we take a picture at night with a flash camera, which of these is the ENERGY CONVERSION that takes place?


A
Chemical------------->Light
B
Light---------------->Sound
C
Mechanical-------------->Light
D
There is no energy conversion

20/07/23

Subject: MATHS
How many whole numbers are there between 1 lakh and 1 crore (excluding both)?
A
9000000
B
9900000
C
9899999
D
10000000

19/07/23

Subject: SCIENCE
Almost all sour fruits contain:
A
Vitamin A
B
Vitamin B
C
Vitamin C
D
Vitamin D

18/07/23

Subject: MATHS
The entire surface of both the solids have to be painted, INCLUDING the bottom faces. If Vijay took 16 minutes to paint the solid he made, how many minutes would Tony take to paint his solid?
(Assume that they paint at the same speed)


A
14
B
16
C
18
D
20

17/07/23

Subject: ENGLISH
Choose from the options a sentence which reports the following sentence correctly.

He said, ''How dare you talk to me like that! Do you know who you are talking to?''
A
He shouted at me and asked how I dared to talk to him like that. He asked whether I knew who I was talking to.
B
He shouted at me and asked how dared I talk to him like that. He asked whether I knew who I was talking to.
C
He shouted at me and asked how I dared to talk him like that. He asked whether I knew who he was taking to.
D
He shouted at me and asked how I dared to talk to him like that. He asked whether he knew who I was talking to.

15/07/23

Subject: SOCIAL STUDIES
In which of these sequences of years was the census of India carried out?
A
2005 – 1985 – 1965
B
2001 – 1991 – 1981 – 1971
C
2001 – 1996 – 1991 – 1986
D
2005 – 1995 – 1985 – 1975

14/07/23

Subject: ENGLISH
Given below are the steps involved in scientific investigation.

1. Collect data or information through observation and other appropriate methods.
2. Identify the hypothesis that can be tested and validated through the experiment.
3. Design the experiment.
4. Perform the experiment and collect data.
5. Analyse the data collected.
6. Validate your hypothesis. If invalid, reformulate the hypothesis and repeat steps 3 to 6.

Your grandfather is ill and goes to see a doctor. The doctor prescribes some medicines after talking to him. However, he wants to have the blood report of your grandfather. Which step in the process of investigation does this correspond to?
A
Step 1
B
Step 3
C
Step 4
D
Step 6

24/06/23

Subject: SOCIAL STUDIES
Shravan is doing a school project on the life of royals of the Mughal era, but is asked not to use any literary sources for information. Which of these could be a source of information on this topic?
A
miniature paintings depicting the Mughal courts and lifestyles
B
murals showing aspects of Mughal society on the mosque walls
C
Islamic sculptures and idols from Mughal monuments
D
black-and-white photographs taken during the Mughal period

20/06/23

Subject: MATHS
p and q are two numbers such that p x q is an integer but p - q is NOT an integer. Which of the following could be the values of p and q?
A
-4 , 4
B
4 , 4
C
{frac(1/4)} , {frac(1/4)}
D
4 , {frac(1/4)}

17/06/23

Subject: SOCIAL STUDIES
Now-a-days most cities of India have Automatic Teller Machines (ATMS). These machines allow people to withdraw cash from their accounts, find out their balance and in some cases also make deposits.

Who among the following can not avail these facilities in ATMs in India?
A
Non-Resident Indians
B
a US citizen
C
a child of Class eight
D
anyone not having a bank account

09/06/23

Subject: SCIENCE
Which of these precautions is NOT relevant to prevent malaria?
A
Pouring a layer of oil on water puddles.
B
Keeping all water containers covered.
C
Covering ourselves properly while sleeping.
D
Washing vegetables before cooking them.

08/06/23

Subject: MATHS
The weighing machine shown below is displaying the weight of a newborn baby.


What is the baby's weight rounded to the nearest 100 grams?
A
2.849 kg
B
2.85 kg
C
2.8 kg
D
3.0 kg

Correct Answer: C

07/06/23

Subject: SCIENCE
Mass is conserved in chemical reactions - specifically, the number of atoms of each element does not change. Equations must therefore be BALANCED. The equation of a reaction between a metal and an acid here, will be balanced when the question mark corresponds to____.
A
2
B
1
C
4
D
0

Correct Answer: B

05/06/23

Subject: ENGLISH
Which of these newspaper headlines is in the active voice and in the present tense?
A
Hurdles Cleared for Kotla ODI
B
Thief Robbed of Cash, Arrested
C
Batsmen Hold Aces in 1st ODI
D
Bowlers Will Need to Keep Their Cool

04/06/23

Subject: SOCIAL STUDIES
Which of these orgnisations is responsible for the conservation of world heritage sites and monuments?
A
B
C
D

Correct Answer: A

04/06/23

Subject: SCIENCE
Air resistance is a special kind of frictional force that acts on objects as they travel through air. Identify the situation where this frictional force is ABSENT, from the ones given below.

A
B
C
D

Correct Answer: A

01/06/23

Subject: MATHS
Arnav used modelling clay to make two cubes whose comparative sizes are shown below.



The volume of clay used for the larger cube is how many times the volume used for the smaller one?
A
12 times
B
8 times
C
4 times
D
2 times

Correct Answer: B

31/05/23

Subject: ENGLISH
A ship has anchored in an ocean. The ocean is 10,500 m deep. An engineer from the ship sent a signal straight down into the water.
How long will it take for the echo to reach him? (velocity of sound in water 1.5 km/s)
A
16.5 sec
B
15 sec
C
14 sec
D
7 sec

Correct Answer: C

30/05/23

Subject: MATHS
Urmil looked at his watch and said, "The number of hours that are left today is exactly one-seventh of the number of hours that have already passed."

What time was Urmil's watch showing?
A
7 AM
B
6:30 PM
C
7 PM
D
9 PM

29/05/23

Subject: ENGLISH
Choose the option with NO spelling error to complete the given sentence.

Children love to watch the __________ light of the crazy-balls which glow in the dark.
A
luminecent
B
lumeniscent
C
luminicent
D
luminescent

Correct Answer: D

27/05/23

Subject: SOCIAL STUDIES
Choose the CORRECT option that MATCHES the sentence given below.

He was known as a great playwright and as a great artist.
A
He was not known only as a great playwright and as a great artist.
B
He was not known as a great playwright but only as a great artist.
C
He was known only as a great playwright and not as a great artist.
D
He was not only known as a great playwright but also as a great artist.

25/05/23

Subject: MATHS
For which of these values of x and y is 2x - 3y = 3x - 2y?
A
x = 1, y = -1
B
x = 2, y = 0
C
x = 0, y = 1
D
x = 1, y = 1

24/05/23

Subject: MATHS
Which of these correctly describes what happens to the molecules in a BOWL OF ICE CREAM when it melts?
A
They absorb heat energy and start moving more slowly.
B
They absorb heat energy and start moving more rapidly.
C
They release heat energy and start moving more slowly.
D
They release heat energy and start moving more rapidly.

Correct Answer: B

22/05/23

Subject: ENGLISH
Which word can precede all the given below words?
an exam
a lesson
a bath
a book
A
read
B
take
C
write
D
do

11/05/23

Subject: MATHS
Frostee ice cream packs are available in two different sizes as shown below.


Abbas needs 10 litres of vanilla ice cream for a party. What is the MINIMUM cost at which Abbas can get enough ice cream for his party?
A
Rs. 340
B
Rs. 350
C
Rs. 400
D
Rs. 450

10/05/23

Subject: SCIENCE
Ravi has three identical looking pieces PQ, RS and TU as shown below. He has been told that some or all of them may be magnets. Others are non-magnets. He conducts the following FOUR experiments.


What can he conclude from this?
A
All are magnets.
B
PQ and RS are magnets, TU is not.
C
PQ and TU are magnets, RS is not known
D
None are magnets.

Correct Answer: B

09/05/23

Subject: MATHS
Study the table given below to answer the question.
Three teams took part in a hockey tournament. Each team played one match against each of the other two teams.
The table below shows the results of the matches, but a part of the table is covered by a stain.


What was the score of the Team X versus Team Y match?
A
Team X - 1, Team Y - 1
B
Team X - 1, Team Y - 0
C
Team X - 2, Team Y - 1
D
Team X - 3, Team Y – 1

Correct Answer : B

08/05/23

Subject: ENGLISH
Who has created this beautiful painting?

Which of these rewrites the above sentence in the Passive Voice correctly?
A
By whom is this beautiful painting created?
B
This beautiful painting had been created by whom?
C
For who is this beautiful painting being created?
D
By whom has this beautiful painting been created?

Correct Answer: D

06/05/23

Subject: SOCIAL STUDIES
If the Prime Minister of India visits the United States to sign an agreement on international cooperation, which of these issues would MOST probably be on the agenda?
A
reforms in Indian Public Sector Undertaking
B
modernisation of Indian Police services
C
strategies to tackle terrorism in both the countries
D
allowing free market economy in Kenya

Correct Answer: C

05/05/23

Subject: SCIENCE
A Vernier scale can be used to obtain more accurate measurements than is possible with a normal scale. The first Vernier here reads 12.3 mm.



What does the second Vernier read?
A
15.2 mm
B
15.0 mm
C
15.10 mm
D
14.10 mm

Correct Answer: B

04/05/23

Subject: MATHS
Abdul pressed 6 buttons on his calculator and got the result as 27. The buttons that he pressed are shown below (in order). The first button is hidden:



Which button did Abdul press first?
A
B
C
D

Correct Answer: C

03/05/23

Subject: SCIENCE
Shown here is an object whose weight is found out using a spring balance, but with the object dipped in water to different extents as shown. In which of these will it record minimum weight?

A
B
C
D

Correct Answer: C

02/05/23

Subject: MATHS
See the table below showing a relation between x and y.



Which of these is the correct relation between x and y?
A
4x = y
B
{frac(x/y)} = 4
C
xy = 4
D
y = x – 3

Correct Answer: B

01/05/23

Subject: ENGLISH
I was instructed to go to the third room on the left when I entered the building.

Which of these states the above sentence in direct speech?
A
I was told, ''When you enter the building, go to the third room on the left.''
B
''I was told that When you enter the building go to the third room on the left''
C
I was told that ''When you enter the building,'' ''go to the third room on the left.''
D
I was told ''when you entered the building go to the third room on the left''

Correct Answer: A

29/04/23

Subject: SOCIAL STUDIES
The temple 'prasadam' of South India and Orissa consist of ingredients that are locally grown for thousands of years. Which of these vegetables/fruits is used for the prasadam?
A
cauliflower
B
banana
C
tomato
D
apple

Correct Answer: B

28/04/23

Subject: SCIENCE
The diagrams show different trials Abdul carried out with carts having different-sized wheels. He started them from different heights and the blocks he put in them were of equal mass. He wants to test this idea: The heavier a cart is, the greater its speed at the bottom of a ramp. Which three trials should he compare?

A
G, T, and X
B
O, T, and Z
C
R, U, and Z
D
S, T, and U

Correct Answer: D

27/04/23

Subject: MATHS
What is the minimum number of degrees by which the shape below has to be rotated (about its centre) so that it looks exactly the same as it does now?

A
90
B
1000
C
1800
D
3600

Correct Answer: D

26/04/23

Subject: SCIENCE
What is the role of GEARS in a car?
A
to start the engine.
B
to ignite the fuel as required
C
to stop the car quickly
D
to transmit power to the wheels as required.

25/04/23

Subject: MATHS
The shape given below is made up of identical small squares.


If the area of the whole shape is 24 cm2, what is its perimeter?
A
24 cm
B
28 cm
C
48 cm
D
56 cm

19/04/23

Subject: SCIENCE
Which BEST describes the surface of the Earth over billions of years?
A
A flat surface is gradually pushed up into higher and higher mountains until it is covered with mountains.
B
High mountains gradually wear down until most of the Earth is at sea level.
C
High mountains gradually wear down as new mountains are continuously being formed, over and over again.
D
High mountains and flat plains stay side by side for billions of years with little change.

18/04/23

Subject: MATHS
The number of pencils in Ketan's box is 6 more than the number of rulers in it. If the number of pencils in his box is P and the number of rulers R, which of the following is true?

A
R + 6 = P
B
P + 6 = R
C
6R = P
D
6P = R

Correct Answer: A

17/04/23

Subject: ENGLISH
Which word has the same meaning as the underlined word in this sentence given below?

He led a humdrum life.
A
isolated
B
exciting
C
monotonous
D
reserved

Correct Answer: C

15/04/23

Subject: SOCIAL STUDIES
Towards the first half of the 20th century, the British government settled the age old land-owning issues in India through the 'Land Reforms Act'. According to this act, which kind of land was transferred from the wealthy feudal lords to the common man?
A
Industrial land
B
Agricultural land
C
Pasture land
D
all kinds of land

Correct Answer: B

14/04/23

Subject: SCIENCE
An instrument attached to a toy car made a punch every second on a strip of paper as the car moved on a smooth surface. This is how the tape looked.

What is the speed of the car between the points A and B?
A
1cm/s
B
8cm/s
C
40cm/s
D
20cm/s

Correct Answer: A

13/04/23

Subject: MATHS
A wall hanging that Jayati sold in the school funfair for Rs 20 had cost her Rs 25 to make. Jayati made a ___________ on the wall hanging.
A
profit of 80%
B
loss of 5%
C
loss of 20%
D
loss of 25%

Correct Answer: C

12/04/23

Subject: SCIENCE
Hajira suffers from acidity in her stomach. Which of these could the medicine prescribed contain?
A
Soda
B
Magnesium Hydroxide
C
Hydrochloric Acid
D
Sodium Chloride

Correct Answer: B

11/04/23

Subject: MATHS
A pair of rational numbers, when MULTIPLIED, gives the answer as (-1). The numbers could be
A
(-p) and p
B
(-p) and (-1)
C
(-p ) and 1
D
(-p) and {frac(1/p)}

Correct Answer : D

10/04/23

Subject: ENGLISH
Choose the OPPOSITE of the underlined word in the sentence below.

I never approve of excessive use of imported materials.
A
ordinary
B
common
C
common
D
moderate

Correct Answer: D

08/04/23

Subject: SOCIAL STUDIES
Which of these is traditionally performed in the courtyard of local Hindu temples?
A
Bihu of Assam
B
Kathakali of Kerala
C
Bhangra of Punjab
D
Karma dance of Chhattisgarh

Correct Answer: B

07/04/23

Subject: SCIENCE
Identify the data that has been used to draw this graph.



A
B
C
D

Correct Answer: A

06/04/23

Subject: MATHS
Which of the following is the BEST estimate of the amount of water in the container shown below?



A
0.6 litres
B
1.3 litres
C
1.5 litres
D
1.7 litres

Correct Answer: C

05/04/23

Subject: SCIENCE
Animals are made up of many atoms. What happens to the atoms after an animal has died?
A
The atoms stop moving. - 4 + 4x
B
The atoms recycle back into the environment.
C
The atoms split into simpler parts and then combine to form other atoms.
D
The atoms no longer exist once the animal has decomposed.

Correct Answer: B

04/04/23

Subject: MATHS
(x - 2)(x - 2) + 4x is equal to
A
x2 - 4 + 4x
B
6x + 4
C
x2> + 4 + 4x
D
x2 + 4

Correct Answer: D

03/04/23

Subject: ENGLISH
Which pair of similar-sounding words can be formed from the clues given below?

'Jams are made from these.'
'One does this to the hatchet to be friends again.'
A
flowers - flour
B
berries - buries
C
peaches - pitches
D
peels - peals

Correct Answer: B

01/04/23

Subject: SOCIAL STUDIES
Raghu holds a driving license issued to him at the RTO office in Bangalore. Possessing it allows him to drive his car _______________.
A
only within Bangalore
B
only within Karnataka
C
all over India
D
all over the world

Correct Answer: C

31/03/23

Subject: SCIENCE
On a rainy night, Mohan observed that every time there was lightning and thunder, he would usually SEE the lightning a few seconds BEFORE he HEARD the thunder. What could be the reason for this?
A
Thunder travels faster than lightning.
B
Sound waves travel faster than light in air.
C
Light waves travel faster than sound waves.
D
Light waves travel through a less dense medium than sound waves.

Correct Answer: C

30/03/23

Subject: MATHS
A part of quadrilateral EFGH is covered with an object X. Which of the following is DEFINITELY true about quadrilateral EFGH?

A
It must be a parallelogram
B
It must be a rectangle
C
It must be a square
D
none of the above

Correct Answer: D

29/03/23

Subject: SCIENCE
The students of a class were asked to look for fossils. Given below are four different rocks and how they are formed.


Which of the rocks is most likely to contain fossils?
A
P
B
Q
C
R
D
S

Correct Answer: A

28/03/23

Subject: MATHS
Ananth ran a 4 km stretch at an average speed of 10 km an hour. If he started his run at 5:00 AM, at what time did he finish?
A
5:24 AM
B
5:40 AM
C
7:25 AM
D
7:30 AM

27/03/23

Subject: ENGLISH
Which of these is the correct form of reported speech of the given statement?

She said to her friend, ''What will you wear for the party tonight?''
A
She told her friend what would she wear for the party tonight.
B
She said to her friend what she would wear for the party that night.
C
She said to her friend what will she wear for the party in the night.
D
She asked her friend what she would wear for the party that night.

25/03/23

Subject: SOCIAL STUDIES
In the beginning of the 20th century, the British divided the Madras, Bengal and Bombay presidencies into a number of smaller states (which we know today as Maharashtra, Kerala, Karnataka etc.) This division was on the basis of _________.
A
religion
B
economy
C
language
D
climate

24/03/23

Subject: SCIENCE
70ml of water was allowed to freeze in a 100ml container.



What will the container look like after the water has frozen?
A
B
C
D

22/03/23

Subject: SCIENCE
A typical displacement reaction can be written as follows: AB + C ----> A + BC

Which among the following is a displacement reaction?
A
H2SO4 + Fe ------------> FeSO4 + H2
B
2H2 O----------------> 2H2 + O2
C
H2 + Cl2--------------> 2HCl
D
CO2, + H2O -------------------->H2CO3

21/03/23

Subject: MATHS
In the figure, lines PT and RU are parallel. What is the measure of the angle marked x?

A
770
B
830
C
970
D
1030

Correct Answer: C

20/03/23

Subject: ENGLISH
Which of these sentences in direct speech is punctuated correctly?
A
''Look, they are coming towards us, let's make sad faces to win their compassion,'' said Vinita.
B
''Look. They are coming towards us. Lets make sad faces to win their compassion.'' said Vinita.
C
''Look! They are coming towards us. Let's make sad faces to win their compassion,'' said Vinita.
D
''Look! they are coming towards us. lets make sad faces to win their compassion.'' said Vinita.

Correct Answer: C

19/03/23

Subject: SOCIAL STUDIES
The 1990s witnessed a countrywide agitation against the implementation of reservation policy in the government and public sector jobs as suggested by the Mandal Commission. The reservation was for __________.
A
all poor children
B
children of politicians
C
certain backward communities
D
Bangaladeshi refugees

Correct Answer: C

17/03/23

Subject: SCIENCE
A girl found the skull of an animal. She did not know what the animal was but she was sure that it preyed on other animals for its food. What clue led to this conclusion?
A
The eye sockets faced sideways.
B
The skull was much longer than it was wide.
C
Four of the teeth were long and pointed.
D
The jaws could move sideways as well as up and down.

Correct Answer: C

16/03/23

Subject: MATHS
On which number line is the position of 3.508 shown most accurately?
A
B
C
D

Correct Answer: A

15/03/23

Subject: SCIENCE
Four school bags of the same size and dimensions but with different strap sizes are available. Among these, the strap offering the most comfort while carrying the bag would be __________________.
A
a 2 cm wide strap
B
a 3 cm wide strap
C
a 4 cm wide strap
D
a 5 cm wide strap

Correct Answer: D

08/03/23

Subject: SCIENCE
Shown below is the range of pH values and its significance.





The strongest acid among the substances listed above is
A
acetic acid
B
sulphuric acid
C
ammonium hydroxide
D
hydrochloric acid

07/03/23

Subject: MATHS
The solid shown below is made of 10 small cubes of side 2 cm each. What is the total volume of the solid?


A
20 sq cm
B
20 cu cm
C
40 sq cm
D
80 cu cm

Correct Answer: D

06/03/23

Subject: ENGLISH
Choose the option with NO spelling error to complete the given sentence.

Children love to watch the __________ light of the crazy-balls which glow in the dark.
A
luminecent
B
lumeniscent
C
luminicent
D
luminescent

Correct Answer: D

04/03/23

Subject: SOCIAL STUDIES
The collage shown here is for a tourism promotion campaign called ‘Incredible India'.



Which of these captions would justify this collage?
A
'Naturally Nepal - Once Is Not Enough'
B
'Karnataka - India's Theatre of Inspiration'
C
'Northeast India - Paradise Unexplored'
D
'Andhra Pradesh - The Kohinoor of India'

Correct Answer: C

03/03/23

Subject: SCIENCE
Which of the following process/processes help in INCREASING the O2 content in air?
A
respiration
B
photosynthesis
C
burning of fuels
D
all of them.

Correct Answer: B

02/03/23

Subject: MATHS
Which of the following could be 110%?
A
The discount on a bag costing Rs. 1500
B
The discount on a bag costing Rs. 1500
C
The percentage of students who passed in an exam
D
The percentage of water in a solution of milk and water.

01/03/23

Subject: SCIENCE
Listed here is the reactivity of certain metals.



Which of the above metals are likely be obtained in their pure states from the earth's crust?
A
gold only
B
sodium only
C
gold and platinum
D
zinc and sodium

Correct Answer: C

28/02/23

Subject: MATHS
The integer that is 25 more than (-435) is
A
-450
B
-460
C
-410
D
460

Correct Answer: C

27/02/23

Subject: ENGLISH
Which word can precede all the given below words?
an exam
a lesson
a bath
a book
A
read
B
take
C
write
D
do

Correct Answer: B

25/02/23

Subject: SOCIAL STUDIES
Which of these months is the official year-ending for all financial institutions of the country, including banks?
A
January
B
October
C
March
D
December

Correct Answer: C

24/02/23

Subject: SCIENCE
Given here is a table that shows the hardness of certain objects.



When two objects are rubbed against each other, the one with a higher hardness value usually makes a mark or scratch on the one with the lower hardness value.

On which of these can a blade produce a scratch?
A
Diamond
B
Quartz, diamond
C
All objects other than diamond
D
Finger nail and talc

Correct Answer: D

23/02/23

Subject: MATHS
Which of the following is the square of a whole number?
A
800
B
903
C
1369
D
4000

Correct Answer : C

22/02/23

Subject: SCIENCE
An apartment with ten floors has a water supply design as shown in the figure.
If three people on the 1st, 5th and 10th floors open their taps at the same time, where will the pressure be the maximum?



A
the tenth floor
B
the fifth floor
C
the first floor
D
all the floors will have the same water pressure.

Correct Answer: C

21/02/23

Subject: MATHS
Four people P, Q, R and S have to sit on the four stools marked 1, 2, 3 and 4.



P goes and sits down on stool 4. Q does not want to sit on a stool that is immediately next to P's. Keeping this in mind, in how many different ways could Q, R and S arrange themselves on the remaining stools?
A
6
B
4
C
3
D
2

Correct Answer: B

20/02/23

Subject: ENGLISH
He worked night and day. He achieved success.

Which option combines the two sentences above without changing its meaning?
A
He achieved success and worked night and day.
B
He achieved success though he worked night and day.
C
Having worked night and day, he achieved success.
D
Having achieved success he worked night and day.

Correct Answer: C

18/02/23

Subject: SOCIAL STUDIES
Expressive 'mudras', 'abhinaya' and 'asanas' together characterise a good _________.
A
classical dancer
B
modern sculpture
C
mural painter
D
folk singer
Correct Answer: A

17/02/23

Subject: SCIENCE
About how much would a jerry-can (a type of container) having 10 litres of water weigh?
A
100 grams
B
1 kilogram
C
10 kilograms
D
20 kilograms

Correct Answer: C

16/02/23

Subject: MATHS
How many 3 × 3 squares looking exactly like the one shown next to the board can you find on the chessboard?



A
18
B
10
C
9
D
6

Correct Answer: A

15/02/23

Subject: SCIENCE
Ashok has an aquarium at home. He wants to increase the oxygen available to his fish. What should Ashok do?
A
add some more fish to his aquarium
B
add some water plants
C
reduce the light in the aquarium
D
add some shells and stones into the aquarium

Correct Answer: B

14/02/23

Subject: MATHS
Venu has a rectangular sheet of cardboard:



With a single straight cut, he cuts the rectangular sheet into two pieces.
The total number of corners on the two resulting pieces COULD be
A
4
B
5
C
7
D
10

Correct Answer: C

13/02/23

Subject: ENGLISH
Who has created this beautiful painting?

Which of these rewrites the above sentence in the Passive Voice correctly?
A
By whom is this beautiful painting created?
B
This beautiful painting had been created by whom?
C
For who is this beautiful painting being created?
D
By whom has this beautiful painting been created?

Correct Answer: D

11/02/23

Subject: SCIENCE
Black bucks are fast running herbivorous animals and prefer to live in hordes. In what kind of habitat are you likely to find these animals?


A
a thick rain forest
B
an open meadow
C
a desert
D
cold mountain peaks

Correct Answer: B

10/02/23

Subject: SCIENCE
Which of these is an ill-effect of a depleting ozone layer?
A
infection of lungs
B
skin cancer
C
retarded growth
D
forest fires

Correct Answer: B

09/02/23

Subject: MATHS
In the figure shown below, PQRS is a square and TPS is an equilateral triangle. The bisectors of angle PTS and angle QRS meet at point O.
What is the measure of angle TOR?
A
1350
B
1400
C
1450
D
1500

Correct Answer: A

08/02/23

Subject: SCIENCE
Given below is an incomplete food chain. Identify the right animals from the list given to complete the food chain.

A
1-rat, 2-cat, 3-snake, 4-hawk
B
1-rat, 2-snake, 3-cat, 4-hawk
C
1-rat, 2-hawk, 3-snake, 4-cat
D
1-rat, 2-hawk, 3-cat, 4-snake

Correct Answer: A

07/02/23

Subject: MATHS
Ginny has a set of 6 small cubes of the same size with designs painted on the faces. No two faces (among ALL 6 cubes) have the same design.

A

B
C
D

Correct Answer: C

06/02/23

Subject: ENGLISH
I was instructed to go to the third room on the left when I entered the building.

Which of these states the above sentence in direct speech?
A
I was told, ''When you enter the building, go to the third room on the left.''
B
''I was told that When you enter the building go to the third room on the left''
C
I was told that ''When you enter the building,'' ''go to the third room on the left.''
D
I was told ''when you entered the building go to the third room on the left''

Correct Answer: A

04/02/23

Subject: SOCIAL STUDIES
The picture below shows an initiative by ITC limited as described in the caption. It is an example of

A
Corporate Social Responsibility
B
Diversification
C
Mergers and Acquisition
D
Market Research

Correct Answer: A

03/02/23

Subject: SCIENCE
Plants have unique features that help them fit into the environment they live in. Which among these is NOT an illustration of the above statement?
A
Lotus leaves have a wax like coating on their upper surface
B
A submerged water plant hydrilla has finger like leaves.
C
Money plant is a climber.
D
Cactus has leaves reduced in size.

Correct Answer: C

02/02/23

Subject: MATHS
Lallu deposits Rs.10000 in a local bank where the annual (simple) interest rate is 6%. For how many years will he have to keep the money there if he wants to have Rs. 13000 in his account?
A
3
B
5
C
6
D
21

Correct Answer: B

01/02/23

Subject: SCIENCE
Which among these is NOT an alternative energy source?
A
Wind
B
Sun
C
Water
D
Natural gas

Correct Answer: D

31/01/23

Subject: MATHS
What is the least number of degrees by which figure X has to be rotated about its centre to get figure Y?

A
600
B
450
C
300
D
150

Correct Answer: C

30/01/23

Subject: ENGLISH
Which of these rewrites the given sentence in its NEGATIVE FORM without changing the meaning?

He only said and did what he sincerely believed.
A
He said and did nothing but what he sincerely believed.
B
He only said and did everything that he sincerely didn't believe.
C
He only said and did nothing that he did not want to believe.
D
He only said and didn't do what he sincerely didn't believe.

Correct Answer: A

27/01/23

Subject: SCIENCE
Ravi has suspended a bar magnet above his desk. The desk is in front of a window, through which the rising sun can be seen directly. In which orientation will the magnet come to rest?

A

B
C
D

26/01/23

Subject: MATHS
A certain common formula converts the numbers in Box 1 to the corresponding ones in Box 2. Another formula then converts the numbers in Box 2 to the corresponding ones in Box 3:


If we wanted to convert a number m in Box 1 directly to the corresponding number in Box 3, we could use the formula
A
3m + 2
B
3m x 3
C
3(m + 2)
D
m + 2 x 3

Correct Answer: C

25/01/23

Subject: SCIENCE
Which of these organs is NOT a part of our digestive system?
A
endocrine glands
B
salivary glands
C
oesophagus
D
liver

Correct Answer: A

24/01/23

Subject: MATHS
x and y are two numbers such that 0 < x < 1 and 1 < y < 2. Which of the following COULD be an integer?
A
x + y
B
y - 1
C
x + 2
D
y

Correct Answer: A

23/01/23

Subject: ENGLISH
Choose the option that completes the given sentence meaningfully.

The man was as cool as a ________ in the midst of the chaos surrounding him.
A
sand
B
apple
C
cucumber
D
coconut

Correct Answer: C

22/01/23

Subject: SCIENCE
How will the figure shown here look if seen through the rear view mirror of a car?


A

B
C
D

19/01/23

Subject: MATHS
1 millilitre (ml) can be defined as the volume of liquid that a cubical box of side 1 cm can hold.
How many LITRES of liquid can the container shown here hold?


A
2000
B
20
C
10
D
2

Correct Answer: D

18/01/23

Subject: SCIENCE
Which of these is made with the help of bacteria?
A
curd (dahi in Hindi)
B
cream
C
soap
D
cooking oil

Correct Answer: A

17/01/23

Subject: MATHS
Hema has weighing scales with a removable pan. One day, she adjusts it as shown below and then weighs a bag of flour:



How much did the bag of flour weigh?
A
1 kg 50 g
B
1 kg 400 g
C
1 kg 500 g
D
1 kg 600 g

Correct Answer: B

16/01/23

Subject: ENGLISH
The crowded bus, bursting at every stage, crawled heavily to a stop with a groan.

What is the figure of speech used in the sentence above?
A
simile (expresses resemblance between things of different kinds usually using 'like' or 'as')
B
metaphor (expression used to refer to something it does not literally denote)
C
alliteration (same starting sound in different words of a line)
D
personification (attributing human characteristics to inanimate objects)

Correct Answer: D

14/01/23

Subject: SCIENCE
The Government of India wants to set up a commission for enquiry into the crime rate in Mumbai. Which ministry will this commission come under?
A
Ministry of Human Resources Development
B
Ministry of Defence
C
Ministry of Home
D
Ministry of Urban Development

Correct Answer: C

13/01/23

Subject: MATHS
The atomic number of oxygen is 8. This means that oxygen has 8 positively charged particles called protons in its nucleus. An atom does not possess any charge. What is the number of electrons (negatively charged particles) in an oxygen atom?
A
0
B
2
C
8
D
12

Correct Answer: C

12/01/23

Subject: MATHS
2x(x - 2) - 2(x - 1) is equal to
A
2x2+ 6x - 2
B
2x2 - 4x - 2
C
2x2 - 6x + 2
D
2x2 - 4x – 1

Correct Answer: C

11/01/23

Subject: SCIENCE
Which of the following organs is made up of involuntary muscles?
A
finger
B
heart
C
thumb
D
tongue

Correct Answer: B

10/01/23

Subject: MATHS
A parallelogram MNOP will be a rectangle when diagonals MO and NP
A
are equal
B
are perpendicular
C
bisect each other
D
intersect at an angle of 450

Correct Answer: A

09/01/23

Subject: ENGLISH
Choose the sentence which has been punctuated correctly.
A
The teacher asked the students, have you finished your work?
B
The teacher asked the students, ''Have you finished your work?''
C
The teacher asked the students have you finished your work?
D
The teacher asked the students that, ''Have you finished your work?''

Correct Answer: B

07/01/23

Subject: SOCIAL STUDIES
Which of the following buildings depict a fusion of Islamic, Rajput and European architecture forms?
A
Victoria Memorial in Kolkata
B
Rashtrapati Bhavan in New Delhi
C
Ajanta Caves in Maharashtra
D
Charminar in Hyderabad

Correct Answer: B

06/01/23

Subject: SCIENCE
Identify the list below that contains only INVERTEBRATES.
A
crabs, turtles, tortoises, whales.
B
fish, spiders, men, monkeys
C
sponges, scorpions, crabs.
D
centipedes, earthworms, crocodiles

Correct Answer: C

05/01/23

Subject: MATHS
25% of n = 50 % of
A
n⁄2
B
n
C
2n
D
25n

Correct Answer: A

04/01/23

Subject: SCIENCE
Anusha observed some green patches on an orange peel she had thrown into the dustbin three days earlier. This green patch is most likely to be
A
a kind of bacteria
B
a kind of algae
C
a kind of fungus
D
a kind of virus

03/01/23

Subject: MATHS
Which digit would appear in the TEN THOUSANDTHS place in the decimal form of 5⁄6?
A
0
B
2
C
3
D
8

02/01/23

Subject: ENGLISH
Choose the sentence which is grammatically correct.
A
She will, as soon as your back is turning, repeat it to all the others.
B
She will, as soon as your back is turning, repeat it to all others.
C
She will, as soon as your back is turned, repeat it to all the others.
D
She will, as quick as your back is away, repeat it to all the others.

31/12/22

Subject: SOCIAL STUDIES
The stamp shown below was issued by

A
a modern Indian state government
B
the Government of India
C
a princely state
D
a city municipal corporation

30/12/22

Subject: SCIENCE
A mirror is placed along the dotted line on the letter N. In which position will the image in the mirror coincide EXACTLY with the other half of the letter?

A

B
C
D

29/12/22

Subject: MATHS
If x - y + z = 0, then y equals
A
x - z
B
z + x
C
-x+z
D
-x_z

28/12/22

Subject: SCIENCE
The diagram represents a mixture of nitrogen and hydrogen molecules in a closed container. Assume that the number of atoms/molecules is exactly what is shown in the diagram.


Which diagram shows the results after the mixture reacts as completely as possible according to the equation: N2 + 3H2> 2NH3
A

B
C
D

27/12/22

Subject: MATHS
A number line is shown below with a point P marked on it:


Which of the following points will be the furthest from P when marked on the number line?
A
-1⁄5
B
1⁄4
C
-8⁄7
D
-7⁄3

26/12/22

Subject: ENGLISH
In which of these can 'since' be replaced by 'because' without changing the meaning of the sentence?
A
I have enjoyed studying science since I was a child.
B
I am interested in this novel since I like mystery stories.
C
Since time immemorial, man has been worshipping nature.
D
I've been waiting for my friend since 5 o'clock.

24/12/22

Subject: SOCIAL STUDIES
South Africa was one of the last European colonies to become a free democratic republic in recent times. Which of these was probably the main attraction of the region to European colonists?
A
its alluring spices
B
its rich forests
C
its gold and diamond mines
D
its English speaking people

23/12/22

Subject: SCIENCE
Sridevi choose four similar leaves of a plant. On leaf 1 she smeared petroleum jelly on both surfaces, on leaf 2 she smeared petroleum jelly on the upper surface and on leaf 3 she smeared petroleum jelly on the lower surface. Which of the following should she do to leaf 4 if she wants to find out whether transpiration takes place ONLY from the lower side of the leaf?
A
She should smear petroleum jelly on both sides.
B
She should smear petroleum jelly on only the upper surface.
C
She should smear petroleum jelly on only the lower surface.
D
She should not smear any petroleum jelly at all on either side.

22/12/22

Subject: MATHS
If the measure of one angle of an isosceles triangle is 400, another angle of the triangle CANNOT measure
A
400
B
700
C
800
D
100

21/12/22

Subject: SCIENCE
On earth, an ice cube floats in water with 9/10 its volume submerged as shown in the figure.



If the experiment were to be done in a lunar module parked on the lunar surface, the ice cube would
A
float with the same volume immersed in water
B
float with its entire volume immersed in water
C
float or sink depending on the volume of the ice cube
D
sink because the gravity on the moon is less than that of the earth

20/12/22

Subject: MATHS
If a number is decreased by 20%, it becomes
A
four fifths of itself
B
two fifths of itself
C
one fifth of itself
D
one eighth of itself

19/12/22

Subject: ENGLISH
Which of these newspaper headlines is in the Active Voice?
A
'Secret Military Codes Stolen'
B
'A Robot That can Respond Designed'
C
'Asian Jailed for Six Months for Bomb Hoax'
D
'HC Rejects Appeal for Another Entrance Test'

17/12/22

Subject: SOCIAL STUDIES
The Assam-Bengal Company runs a river cruise on the Brahmaputra River within Assam. If you travel on this cruise, which of these is NOT going to be a sight during the sail?
A
shifting banks of river and huge piles of silt
B
small fishing boats and crowded ferries
C
rich grassland and forest lands across its banks
D
lush mangroves and estuarine forests on its banks

16/12/22

Subject: SCIENCE
For several centuries it was believed that the earth was the centre of the universe and the stars and planets revolved around it. Which of these is likely to be the reason scientists like Galileo were convinced that this was not correct, and the earth actually revolved around the sun?
A
They looked through telescopes and saw that the earth was actually revolving around the sun.
B
They found that natural phenomena (like the phases of Venus) could be explained better this way.
C
The church in 17th century advocated it based on passages in the Holy Scriptures.
D
They found that the majority of scientists had a similar idea, and therefore believed that this must be correct.

15/12/22

Subject: MATHS
2.36 × 0.065 = 0.1534

Which of the following will have a value close to 15?
A
0.236 × 0.0065
B
2.36 × 0.0065
C
23.6 × 0.65
D
236 × 6.5

14/12/22

Subject: SCIENCE
Plants produce glucose from carbon dioxide and water during photosynthesis. Glucose and its derivatives are the building materials of plants.
From where does the majority of the bulk of a tree come?
A
the soil
B
the air
C
the minerals + chlorophyll
D
the water + minerals

13/12/22

Subject: MATHS
Of the pieces shown here, which two could form a semicircle when placed next to each other with their edges touching?

A
Z and W
B
W and Y
C
Y and Z
D
Z and X

12/12/22

Subject: ENGLISH
Which of these rewrites the given sentence correctly in direct speech?

The old man exclaimed with sorrow that his only son had died.
A
The old man said, ''My only son has passed away.''
B
The old man said, ''Oh my only son had passed away.''
C
The old man said, ''Alas! My only son has died.''
D
The old man said, ''Alas, my only son has died''

10/12/22

Subject: SOCIAL STUDIES
The following is an imaginary dialogue between two characters in a historical novel.
'Maharaja, I grew some 100 maunds of Jawar in my field last year. But most of these were taken as land tax - 50 maunds by the amil, 14 maunds by the zamindar and 10 maunds by the village moneylender. And now this year a new tax is being imposed. How will we pay all these taxes and what will we eat?' The dialogue suggests:
A
exploitation of farmers
B
exploitation of moneylenders
C
exploitation of zamindars
D
exploitation of maharajas

09/12/22

Subject: SCIENCE
Environmentalists recommend an increased use of biofuels to reduce pollution. Identify a biofuel from the following list.
A
ethanol
B
coal
C
kerosene
D
diesel

08/12/22

Subject: MATHS
24 + 32 is equal to
A
14
B
17
C
24
D
25

07/12/22

Subject: SCIENCE
Given below are the major food components of 100 grams of a particular brand of wheat flour. The energy released per gram of each food component is also specified in the table.


Based on the given information, calculate the approximate amount of energy in kcal released by 100 grams of the wheat flour.
A
300
B
310
C
350
D
360

08/12/22

Subject: MATHS
10500000 can also be written as
A
10.5 lakhs
B
1 crore 5 lakhs
C
1 crore 50 lakhs
D
105 thousand

05/12/22

Subject: SCIENCE
Choose the option which means the same as the given statement.

As you sow, so shall you reap.
A
Good actions will bring rewards; bad ones, punishment.
B
We must be thankful for all that we get in life.
C
Seeds should be planted at the right time.
D
We should take advantage of every opportunity we get.

03/12/22

Subject: SOCIAL STUDIES
Which of these farming methods in a hill slope is ideal for rice cultivation?
A
B
C
D

02/12/22

Subject: SCIENCE
An object balances itself when a perpendicular line drawn through its Centre of Gravity (CG), passes through its base.

See the picture of a popular toy given below. The horse, the rider and the ball are all joined together as one single unit. This entire unit is balanced on the thin black rod shown. At which of the indicated positions, could the Centre of Gravity (CG) of the horse, rider and ball unit be?
A
A
B
B
C
C
D
D

01/12/22

Subject: MATHS
Nandini makes 'halwa' one evening and divides it into four equal portions for her family of four. However, just as they are about to eat it, an unexpected guest arrives and Nandini has to now re-divide the halwa into five equal portions. By what percentage has each family member's share reduced due to this?
A
5%
B
10%
C
20%
D
25%

30/11/22

Subject: SCIENCE
The unit used to measure distance or length is metres (m), the unit used to measure time is seconds (s) and the unit used to measure mass is kilogram (kg).


Which of the following will be the units of ''Work done''?
A
kg × m/s
B
s/m2
C
kg × m2/s2
D
kg2/m2 × s

20/11/22

Subject: ENGLISH
Choose the CORRECT option that MATCHES the sentence given below.

He was known as a great playwright and as a great artist.
A
He was not known only as a great playwright and as a great artist.
B
He was not known as a great playwright but only as a great artist.
C
He was known only as a great playwright and not as a great artist.
D
He was not only known as a great playwright but also as a great artist.

17/11/22

Subject: MATHS
Which of the following lists has numbers arranged in INCREASING order?
A
-1/8, -1/7, 1/7, 1/8
B
-1/7, -1/8, 1/7, 1/8
C
-1/8, -1/7, 1/8, 1/7
D
-1/7, -1/8, 1/8, 1/7

16/11/22

Subject: SCIENCE
Which of these best represents the surface of water if its quantity is exactly 40 ml?
A

B
C
D

Correct Answer: B

15/11/22

Subject: MATHS
x and y are two rational numbers.
If x + y < 5 and 3 < x < 5 , which of the following values can y have
A
8/3
B
2
C
0
D
3

Correct Answer: C

14/11/22

Subject: ENGLISH
Here are two words 'fleet' and 'sate'. By removing one letter from the first word and adding it to the second word, and rearranging the letters, we get two words 'feel' and 'taste'.

One can similarly start with the words 'stir' and 'miser' to get which two words?
A
'sit' and 'miserr'
B
'tis' and 'rimers'
C
'sir' and 'mister'
D
'rit' and 'misser'

12/11/22

Subject: SOCIAL STUDIES
Which of these earnings would attract corporate tax in India?
A
the earnings of the National Museum through the sale of tickets
B
the sale of pamphlets to farmers by an NGO
C
earnings through collection of fees by a city school
D
earnings through selling of religious CDs by a music company

Correct Answer: D

11/11/22

Subject: SCIENCE
Prateek has three iron bars (PQ, RS and TU) that look alike. He knows that one or more of them may be magnet, but does not know which. He conducts the following experiments.
In Figure 1, there is attraction between RS and PQ and in Figure 2, there is no attraction between PQ and TU. What can he conclude from these experiments?

A
RS and PQ are magnets and TU is an iron bar
B
RS is a magnet and PQ and TU are iron bars
C
RS and PQ are magnets - he cannot be sure about TU
D
RS is a magnet, PQ is an iron bar - he cannot be sure about TU

Correct Answer: B

09/11/22

Subject: SCIENCE
The room temperature is 20 degrees C. What is the temperature of this ball of wool?

A
less than 20 degrees C
B
equal to 20 degrees C
C
more than 20 degrees C
D
depends on the thickness of the woollen ball

Correct Answer: B

08/11/22

Subject: MATHS
In which of the following cases would the quantity q DECREASE PROPORTIONATELY when the quantity p is INCREASED?
A
p = 3q
B
p - 3 = q
C
p = 3⁄q
D
1⁄p = 3⁄q

Correct Answer: C

07/11/22

Subject: ENGLISH
Identify the sentence which has a word that rhymes with 'reel'.
A
The child ran down the stairs with her hand on the rail.
B
The fleet of ships looked lovely in the pale evening light.
C
A good doctor tries his best to heal even his sickest patient.
D
My friend looked up worriedly as she heard the door bell.

Correct Answer: C

05/11/22

Subject: SOCIAL STUDIES
Over millennia, many communities from abroad have migrated and settled in India. Many of them have adopted Indian customs and languages in their lifestyle. Which language do Parsees, who are one such community, speak?
A
Malayalam
B
Konkani
C
Gujarati
D
Marathi

04/11/22

Subject: SCIENCE
The voltage in the socket in Satish's home is 220V. While traveling on a train, Satish finds an electrical socket marked '110V'. In which of these places can Satish charge his mobile, if the charger is rated as shown below?
A
only at home
B
only in the train
C
neither at home nor in the train
D
both at home and in the train

Correct Answer: D

03/11/22

Subject: MATHS
Convert 3.33% into a fraction.
A
333
B
333/10
C
333/100
D
333/10000

Correct Answer: D

02/11/22

Subject: SCIENCE
Study the conversation between two atoms. Which of the following could be the missing words?

A
a proton
B
an electron
C
a neutron
D
an atom

Correct Answer: B

01/11/22

Subject: MATHS
How long would it take a man to count from 1 to 1 million, if he takes 1 second to count each number, does not skip numbers, and does not take any breaks till he finishes?
A
About five hours
B
About three days
C
About one and a half weeks
D
Almost two years

Correct Answer: C

31/10/22

Subject: ENGLISH
Choose the option that completes the given sentence correctly.

My friend was in search of some reasonable _____________.
A
acomodation
B
accommodation
C
accomodation
D
ackomodation

Correct Answer: B

29/10/22

Subject: SOCIAL STUDIES
The costumes, housing and decorated elephants shown in the festival below suggest that it is being celebrated in



A
Bengal
B
Punjab
C
Kerala
D
Chhatisgarh

Correct Answer: C

28/10/22

Subject: SCIENCE
Study the figure given below carefully. it shows air rising from the sea being replaced by air from the land.



Choose the appropriate row from the table, that BEST represents the time of the day, the type of breeze and the reason the breeze occurs.

A
A
B
B
C
C
D
D

Correct Answer: C

27/10/22

Subject: MATHS
Which of the following will have 20 diagonals if the number of diagonals of an 'n' sided polygon is n(n - 3)⁄2?
A
B
C
D

Correct Answer: D

26/10/22

Subject: SCIENCE
Line up four tennis balls on the table so that they just touch each other as shown below. What will happen to the row of tennis balls if a fifth tennis ball is made to collide into the first one?



A
B
C
D

Correct Answer: B

25/10/22

Subject: MATHS
Sukhia, Babu and Ram are 3 farmers each having a small plot of land. Two of them have rectangular plots, and one has a square plot. Some more facts about their plots are:



Who has the square plot?
A
Sukhia
B
Babu
C
Ram
D
None of them

Correct Answer: B

23/10/22

Subject: ENGLISH
Choose the option that completes the given sentence correctly.

When the car is standing still, it is _____________.
A
stationery
B
stationary
C
stetionary
D
stationnary

Correct Answer: B

22/10/22

Subject: SOCIAL STUDIES
The map shown here represents

A
highway construction project
B
aviation and railway
C
linking of rivers
D
ancient Indian trade routes

Correct Answer: A

21/10/22

Subject: SCIENCE
All acids MUST contain
A
hydrogen
B
hydrogen and oxygen
C
hydrogen, oxygen and any other element like sulphur
D
(It all depends on the type of acid.)

Correct Answer: A

20/10/22

Subject: MATHS
What is the area of the shaded portion if the area of the circle is 56 sq cm?
A
12 sq cm
B
14 sq cm
C
14.8 sq cm
D
18.5 sq cm

Correct Answer: B

19/10/22

Subject: SCIENCE
Study the given food chain carefully and answer the following question. One of these organisms is an important part of the food web, but is not either a producer, primary consumer, secondary consumer or tertiary consumer. Which organism?
A
algae
B
bacteria
C
snail
D
fish

Correct Answer: B

18/10/22

Subject: MATHS
Bablu has three sticks of lengths 3, 4 and 5 cm.
Ramesh has three sticks of lengths 3, 5 and 9 cm.
How many triangles can Bablu and Ramesh make with their sticks?
A
Bablu: 1, Ramesh: 0
B
Bablu: 2, Ramesh: 1
C
Bablu: 1, Ramesh: 1
D
Bablu: 0, Ramesh: 2

Correct Answer: A

17/10/22

Subject: ENGLISH
Choose the correct meaning of the given sentence.

All that glitters is not gold.
A
Everything that shines looks as good as gold.
B
Something that looks very good may not be so great.
C
There are more ways to the wood than one.
D
All's well that ends well.

Correct Answer: B

15/10/22

Subject: SOCIAL STUDIES
Mr. Ghosh lives in Kolkata. He is being asked to pay 'Water Tax' by an authority for which he wants a clarification. From which of these should he seek clarification?
A
Kolkata High Court
B
Kolkata Municipal Corporation
C
Writers Building
D
State Pollution Control Board

Correct Answer : B

14/10/22

Subject: SCIENCE
If equal weights of all the three states of a substance X are taken, which state will have the MAXIMUM number of molecules?
A
solid
B
liquid
C
gas
D
(All three will have the same number of molecules.)

Correct Answer: D

14/10/22

Subject: SCIENCE

lf the above pattern of faces continues, what will the 1001st face be?
A
B
C
D

Correct Answer: D

13/10/22

Subject: MATHS

lf the above pattern of faces continues, what will the 1001st face be?
A
B
C
D

Correct Answer : D

12/10/22

Subject: MATHS
Which of the following would a person travelling in a car feel if the car were to overtake a moving bus? The person will feel that____________.
A
the bus is going forward
B
the bus is standing still for a moment but later travelling forwards
C
the bus is standing still for a moment but later travelling backwards
D
the bus is going backwards for a moment and then standing still

Correct Answer: C

11/10/22

Subject: MATHS
Which of these is true for any non-zero rational number p?
A
p × (- p) = 1
B
(-p) ÷ p = -1
C
p - (-p) = 0
D
-1⁄p=p

Correct Answer: B

10/10/22

Subject: ENGLISH
Choose the correct meaning of the underlined words in the sentence below.

Somehow an Italian model just didn't fit the bill.
A
meet the requirements
B
pay the bill
C
fit into the billboard
D
have enough money

Correct Answer : A

08/10/22

Subject: SOCIAL STUDIES
Kalarippayattu is an ancient dance form of Kerala which is still practised in the region. The dance depicts martial art forms. Which of these is likely to be Kalarippayattu?
A
B
C
D

Correct Answer: B

07/10/22

Subject: SCIENCE
Identify the name of the disease that could complete this passage correctly.

----------------------can be spread through contact with infected faeces or through infected droplets travelling through the air, in food, or in water. 10% of infected people develop symptoms and 1% develop the paralytic form of this disease.
A
cholera
B
diarrhoea
C
jaundice
D
polio

Correct Answer: D

06/10/22

Subject: MATHS
Latif's Maths teacher asks him to think of any whole number from 1 to 10. She then asks him to do the following:

1. Double this number.
2. Add 4 to the result.
3. Multiply the sum by 5.

If we call the number Latif thinks of N, the number he gets at the end will be
A
(N + 2) × 10
B
N + 4 × 2 × 5
C
(N × 2) + (4 × 5)
D
10N + 4

Correct Answer: A

05/10/22

Subject: SCIENCE
In a concave mirror, I can see my face completely. From this I can say that my face must be _____________.
A
between the pole of the mirror and the focus of the mirror
B
at the focus of the mirror
C
between the focus of the mirror and centre of curvature of the mirror
D
very far from the mirror

Correct Answer: A

04/10/22

Subject: MATHS
If the area of a circle of radius 7 cm is 154 sq cm, what is the area of the shaded portion of the figure shown below?


A
9 sq cm
B
10.5 sq cm
C
12.25 sq cm
D
16.5 sq cm

Correct Answer: B

03/10/22

Subject: SCIENCE
Choose the option that completes the given sentence meaningfully.

The man was as cool as a ________ in the midst of the chaos surrounding him.
A
sand
B
apple
C
cucumber
D
coconut

Correct Answer: C

01/10/22

Subject: SOCIAL STUDIES
It was controlled at different times by Portugal, Netherlands and the UK and has India's oldest synagogue (Jewish temple). Its population comprises Hindus, Muslims, Christians, Sikhs and others and it houses India's Southern Naval Command. Identify this town.
A
Chennai
B
Kochi
C
Surat
D
Diu

Correct Answer: B

30/09/22

Subject: SCIENCE
Four friends from class 8 were talking about the moon.
Ramnath: In the daytime on some days, we can see the moon

Kaushik: In the daytime, the moon is on the other side of the Earth

Chiranjivi: On all new moon nights, the Earth blocks the moon

Ratna: The moon does not have its own light.,
Which of them is/ are correct?
A
only Kaushik and Chiranjivi
B
only Ramnath and Ratna
C
only Kaushik, Chiranjivi and Ratna
D
only Ratna

Correct Answer: B

29/09/22

Subject: MATHS
One of these numbers is a prime number. Which one could it be?
A
211 + 5
B
211 + 25
C
211 + 5
D
(211)5

Correct Answer: A

28/09/22

Subject: SCIENCE
According to Aristotle, long-range forces did not exist. Long-range forces are those that can act from a distance, without being in contact with the body. Which of these would have EXISTED according to him?
A
a body falling to the ground when thrown up
B
like poles of two magnets repelling each other
C
the slowing down of a car due to application of brakes
D
a rubbed plastic comb attracting small pieces of paper

Correct Answer: C

28/09/22

Subject: SCIENCE
According to Aristotle, long-range forces did not exist. Long-range forces are those that can act from a distance, without being in contact with the body. Which of these would have EXISTED according to him?
A
a body falling to the ground when thrown up
B
like poles of two magnets repelling each other
C
the slowing down of a car due to application of brakes
D
a rubbed plastic comb attracting small pieces of paper

Correct Answer: C

27/09/22

Subject: MATHS
Bipasha can eat 24 rasgullas in 12 minutes. Her friend Sujata takes DOUBLE the time to eat the same number.

One day they buy 24 rasgullas and start eating them. If they eat at their normal speeds till the rasgullas are over, how many rasgullas would Sujata have eaten?
A
6
B
8
C
12
D
16

Correct Answer: B

26/09/22

Subject: ENGLISH
Which of these rewrites the given sentence correctly?

The jasmine is one of the most fragrant flowers.
A
No other flower is fragrant like the jasmine.
B
None of the other flowers are as fragrant as the jasmine.
C
The jasmine is more fragrant than any other flower.
D
The jasmine is more fragrant than most other flowers.

COrrect Answer: D

24/09/22

Subject: SOCIAL STUDIES
Which of these administrative units of a state are arranged correctly from LARGER TO SMALLER in size?
A
Districts - Talukas - Villages
B
Cities - Districts - Talukas
C
Talukas - Districts – Cities
D
Districts - Villages – Talukas

Correct Answer: A

23/09/22

Subject: SCIENCE
Kranti did the following experiment. He took two metal sheets, one of copper and the other of silver, but identical in all other respects. He painted the silver sheet black (X) and the copper sheet white (Y). He mounted the metal sheets and fixed a large nail with the help of a piece of wax on both of them as shown in the figure.


He switched on an electric heater kept midway between the plates. After a few minutes, the nail fixed to the silver plate fell. He concluded that black absorbs heat better than white. Is Kranti's conclusion right or not?
A
Yes, it is. The nail behind sheet X falls before the nail behind sheet Y.
B
Yes, it is but he needs to repeat the same experiment at least 2 more times to be sure.
C
No, it is not. He should have painted the copper sheet black and the silver sheet white.
D
No, it is not. He should have taken both the sheets of the same metal.

Correct Answer: D

22/09/22

Subject: MATHS
Look at the polygons shown below. In each of them, ALL the diagonals drawn from ONE vertex are shown.


From these we can say that the number of diagonals that can be drawn from any one vertex of a 20-sided polygon is
A
20
B
19
C
17
D
16

Correct Answer: C

21/09/22

Subject: SCIENCE
Which of the following is NOT a function of the heart?
A
to take in deoxygenated blood from various parts of the body
B
to purify the deoxygenated blood
C
to pump oxygenated blood to various parts of the body
D
to pump deoxygenated blood to the lungs

Correct Answer: B

19/09/22

Subject: ENGLISH
Choose the sentence which has been punctuated correctly.
A
The teacher asked the students, have you finished your work?
B
The teacher asked the students, ''Have you finished your work?''
C
The teacher asked the students have you finished your work?
D
The teacher asked the students that, ''Have you finished your work?''

16/09/22

Subject: SCIENCE
Simple machines are divided up into six basic types. The door stopper of the type shown alongside is classified as a__________.



A
pulley
B
lever
C
wedge
D
wheel and axle

15/09/22

Subject: MATHS
Amaresh has Rs. 25000 to invest. He has a choice of investing it FOR A YEAR in one of the following schemes:

Scheme 1: gives a return of 4.9% every six months.
Scheme 2: gives a return of 8.7% every year.
Scheme 3: gives a return of 10 paise for every rupee per year.
Scheme 4: gives a return of Rs. 9.75 per 100 rupees per year.

Which scheme will give Amaresh the best returns?
A
Scheme 1
B
Scheme 2
C
Scheme 3
D
Scheme 4

Correct Answer: C

14/09/22

Subject: SCIENCE
Put the following objects in the correct order, starting with the object that is closest to Earth.
A
moon, Sun, clouds, pluto, stars
B
clouds, stars, moon, Sun, pluto
C
clouds, moon, Sun, pluto, stars
D
clouds, moon, Sun, stars, pluto

Correct Answer: C

13/09/22

Subject: MATHS
(-5)6 ÷ (-5)3 equals
A
2
B
-25
C
-125
D
125

Correct Answer: C

12/09/22

Subject: ENGLISH
Choose the sentence which is grammatically correct.
A
She will, as soon as your back is turning, repeat it to all the others.
B
She will, as soon as your back is turning, repeat it to all others.
C
She will, as soon as your back is turned, repeat it to all the others.
D
She will, as quick as your back is away, repeat it to all the others.

Correct Answer: C

08/09/22

Subject: MATHS
Atul's score in the Maths exam is 30% less than his score in the Science exam.
His Science score is 87. His Maths score would be
A
87 - 30
B
87 - 30
C
30% of 87
D
70% x 87

07/09/22

Subject: SCIENCE
See the picture of a regular 60W bulb from different angles. Note the parts marked P, Q, R, S, T and U.
To light the bulb from the mains electric supply, the 2 electric wires must be in contact with:



NOTE: Please do NOT try this except under the guidance of a knowledgeable adult or electrician. A wrong arrangement or carelessness can lead to a short circuit or electric shock.
A
P and Q
B
R and S
C
Q and R
D
T and U

Correct Answer: D

06/09/22

Subject: MATHS
6789678967896789 ÷ 6789 is equal to
A
1111
B
10101
C
1000100010001
D
1000100010001000

Correct Answer: C

05/09/22

Subject: ENGLISH
In which of these can 'since' be replaced by 'because' without changing the meaning of the sentence?
A
I have enjoyed studying science since I was a child.
B
I am interested in this novel since I like mystery stories.
C
Since time immemorial, man has been worshipping nature.
D
I've been waiting for my friend since 5 o'clock.